Cutaneous Manifestations of Systemic Diseaes MCQs

Share Embed Donate


Short Description

Descripción: ETAS MCQ with explanation...

Description

Which of the following is an autosomal recessive disorder characterized by pili torti and deafness? 1

Waardenburg Syndrome

2

Sjorgen-Larsson Syndrome

3

Refsum Disease

4

Bjornstad's Syndrome

5

Cockayne Syndrome Q/Q(M)-473949 Report a Problem

Which of the following is an autosomal recessive disorder characterized by pili torti and deafness? 4

Bjornstad's Syndrome

Waardenburg Syndrome is an autosomal dominant disorder. Sjorgen-Larsson Syndrome is an autosomal recessive disorder characterized by a triad of ichthyosis, di- or tetraplegia, mental retardation. Refsum Disease is an autosomal recessive disorder whose features include deafness, mild ichthyosis in adulthood, and accentuated palmar creases. Bjornstad's Syndrome is an autosomal recessive disorder characterized by pili torti and deafness. Cockayne Syndrome is an autosomal recessive disorder whose features include photosensitivity, bird-headed facies, and "Mickey-Mouse" ears, as well as deafness. Q/Q(M)-473949 Report a Problem A young African American patient presents with anemia and spontaneously appearing leg ulcers over  both lateral and medial malleoli. The most likely diagnosis is: 1

Atherosclerotic disease

2

Sickle cell anemia

3

Factitial dermatitis

4

Lupus erythematosus

5

Trauma

Q/Q(M)-479609 Report a Problem A young African American patient presents with anemia and spontaneously appearing leg ulcers over  both lateral and medial malleoli. The most likely diagnosis is: 2

Sickle cell anemia

Sickle cell anemia should be considered most likely likel y in any young African American with spontaneous leg ulcers. The ulcers are more common in people with severe anemia. Q/Q(M)-479609 Report a Problem What is the most common internal cause of intractable pruritus? 1

Hepatitis C

2

Hypothyroidism

3

Hyperthyroidism

4

Chronic renal failure

5

Internal malignancy 1

Q/Q(M)-482361 Report a Problem What is the most common internal cause of intractable pruritus? 4

Chronic renal failure

Chronic renal failure is the most common internal systemic cause of pruritus. Up to 49% of patients with chronic renal failure have pruritus. Other systemic causes of pruritus include liver disease, hepatitis C, hypo and hyperthryoidism, iron deficie ncy anemia, polycyhtemia vera, Hodkin's lymphoma, leukemia, carcinoid, internal malignancy, AIDS, and internal parasites. Q/Q(M)-482361 Report a Problem The peak sensitivity to prophyrins occurs at which wavelengths? 1

220-290nm

2

290-320nm

3

320-400nm

4

400-410nm

5

410-450nm Q/Q(M)-477127 Report a Problem

The peak sensitivity to prophyrins occurs at which wavelengths? 4

400-410nm

The Soret band (400-410 nm) is the portion of ultraviolet wavelengths at which most porphyrins are most sensitive. Q/Q(M)-477127 Report a Problem Regarding eruptive xanthomas, which of the following is true? 1

They occur in the setting of familial hyperlipidemia types I, IV, and V

2

They occur in the setting of familial hyperlipidemia types II and III

3

Triglyceride levels are usually below 500mg/dl

4

They are most commonly found on the eyelids

5

They are not related to alcohol consumption Q/Q(M)-479281 Report a Problem

Regarding eruptive xanthomas, which of the following is true? 1

They occur in the setting of familial hyperlipidemia types I, IV, and V

Eruptive xanthomas generally occur in patients with triglyceride levels of 2000mg/dl or greater. Associations include poorly-controlled diabetes mellitus, retinoids, estrogens, excessive alcohol consumption (leading to pancreatitis) and familial hyperlipidemias types I, IV and V. Clinically, they appear as crops of firm, non-tender yellowish papules with an er ythematous border. Most commonly, they occur on the extensor surfaces, but they can be diffuse. A reduction in triglycerides and/or tight glucose control usually results in a reduction in the number of lesions. 2

Q/Q(M)-479281 Report a Problem

A 63-year-old male develops small, non-tender, violaceous papules on his dorsal hands, face, ears, and trunk. The patient also develops a destructive arthropathy with finger deformities . What percentage of  patients with this condition has an associated malignancy? 1

20-25%

2

5-10%

3

65-70%

4

50-55%

5

85-90% Q/Q(M)-482240 Report a Problem

A 63-year-old male develops small, non-tender, violaceous papules on his dorsal hands, face, ears, and trunk. The patient also develops a destructive arthropathy with finger deformities . What percentage of  patients with this condition has an associated malignancy? 1

20-25%

This patient has developed multicentric reticulohistiocytosis. Histopathology of the skin lesions will display multinucleated oncocytic giant cells with eosinophilic ground glass cytoplasm. Half of these  patients will develop a mutilating arthritis. 20-25% of patients with multicentric reticulohistiocytosis will develop an associated malignancy. The malignancy is usually a carcinoma, but there is no  predominant type. Q/Q(M)-482240 Report a Problem

Which of the following skin findings is most closely linked to hepatocellular carcinoma as a  paraneoplastic syndrome? 1

Pityriasis lichenoides

2

Pityriasis alba

3

Pityriasis amiantacea

4

Pityriasis rotunda

5

Pityriasis rosea Q/Q(M)-482505 Report a Problem

Which of the following skin findings is most closely linked to hepatocellular carcinoma as a  paraneoplastic syndrome? 4

Pityriasis rotunda

Pityriasis rotunda is an dermatosis that features characteristic discrete, circular, scaly, brown patches on the trunk and extremities. Pityriasis rotunda may be associated with systemic diseases in certain racially  predisposed groups (blacks), and has been linked to hepatocellular carcinoma. Q/Q(M)-482505 Report a Problem 3

All of the following are true regarding incontinentia pigmenti EXCEPT: 1

It is caused by a mutation in the NEMO gene

2

It is an X-linked recessive disorder

3

Inflammation and blistering may be followed by hyperkeratotic, verrucous lesions

4

It is associated with cerebellar ataxia

5

It is associated with coloboma and retinal detachment Q/Q(M)-473965 Report a Problem

All of the following are true regarding incontinentia pigmenti EXCEPT: 2

It is an X-linked recessive disorder

Incontinentia pigmenti is an X-linked dominant disorder caused by a mutation in the NEMO gene. Females only present at birth with linear lesions of inflammation and blistering (stage 1), followed by hyperkeratotic verrucous areas (stage 2), hyperpigmentation (stage 3), and then atrophy (stage 4). Systemic findings include psychomotor retardation, microcephaly, seizures, cerebellar ataxia, coloboma, and retinal detachment. Q/Q(M)-473965 Report a Problem A patient on hemodialysis presents with indurated plaques having a peau d �orange texture on the  bilateral lower legs. Which of the following statements is TRUE? 1

The diagnostic histopathological findings include acanthosis and hyperkeratosis

2

Serum protein electrophoresis should be performed

3

Dapsone will likely be effective treatment

4

The face is usually is affected

5

The palms and soles are usually affected Q/Q(M)-477554 Report a Problem

A patient on hemodialysis presents with indurated plaques having a peau d �orange texture on the  bilateral lower legs. Which of the following statements is TRUE? 2

Serum protein electrophoresis should be performed

This patient likely has nephrogenic fibrosing dermopathy (NFD). NFD is an acquired, idiopathic disorder that occurs in renal disease patients. It resembles scleroderma or eosinophilic fasciitis clinically and scleromyxedema histopathologically. Large areas of indurated skin with fibrotic nodules and plaques develop. The extremities are most commonly involved, followed by the trunk. The face,  palms and soles are almost never involved. Histopathologically, Histopathologically, NFD displays a proliferation of dermal fibroblasts and dendritic cells, thickened collagen bundles, increased el astic fibers, and mucin deposition. Serum protein electrophoresis and immunoelectrophoresis results are negative unlike scleromyxedema, and may be helpful to distinguish the two diseases. NFD is usually a chronic,  progressive condition, and favorable responses responses to medical intervention are anecdotal. Q/Q(M)-477554 Report a Problem 4

A 2 year old child is evaluated for suspected diagnosis of neurofibromatosis. Which of the following diagnostic findings is typically absent on exam in this age group? 1

neurofibromas

2

macrocephaly

3

pigmented iris hamartomas

4

seizures

5

cafe au lait macules Q/Q(M)-482529 Report a Problem

A 2 year old child is evaluated for suspected diagnosis of neurofibromatosis. Which of the following diagnostic findings is typically absent on exam in this age group? 1

neurofibromas

Cutaneous neurofibromas typically appear after puberty. The absence of neurofibromas in young children does not rule out this diagnosis. Pigmented iris hamartomas are more likely finding in young children than neurofibromas. Q/Q(M)-482529 Report a Problem A 50-year old woman with a history of spontaneous pneumothorax develops multiple firm, skin colored lesions on her face and neck over a period of several years. This patient should have periodic surveillance for the development of: 1

Renal Cell Carcinoma

2

Gastric Carcinoma

3

Breast Carcinoma

4

Ovarian Carcinoma

5

Lung Carcinoma Q/Q(M)-482253 Report a Problem

A 50-year old woman with a history of spontaneous pneumothorax develops multiple firm, skin colored lesions on her face and neck over a period of several years. This patient should have periodic surveillance for the development of: 1

Renal Cell Carcinoma

This patient has Birt-Hogg-Dube syndrome, an autosomal dominant defect of the BHD gene that encodes folliculin. This syndrome is characterized by the development of trichodiscomas, fibrofolliculomas, and acrochordons starting around age 30. The patients are also at risk for the development of spontaneous pneumothorax and renal cell carcinoma. Q/Q(M)-482253 Report a Problem The shoulder pad sign has been described in which disease? 1

Dermatomyositis 5

2

Cushings disease

3

Systemic amyloidosis

4

Nephrogenic fibosing dermopathy

5

Systemic lupus erythematosis Q/Q(M)-477433 Report a Problem

The shoulder pad sign has been described in which disease? 3

Systemic amyloidosis

The shoulder pad sign has been described in the setting of late systemic amyloidosis and is due to the direct deposition of amyloid in the deltoid muscles. Q/Q(M)-477433 Report a Problem

One might see all of the following laboratory and clinical abnormalities in cryoglobulinemia associated with hepatitis C virus infection EXCEPT: 1

Elevated liver function tests

2

Positive rheumatoid factor

3

Elevated C3 levels

4

Acrocyanosis

5

Urticarial plaques Q/Q(M)-473927 Report a Problem

One might see all of the following laboratory and clinical abnormalities in cryoglobulinemia associated with hepatitis C virus infection EXCEPT: 3

Elevated C3 levels

Laboratory abnormalities of HCV include an elevation of liver enzymes, positive rheumatoid factor (70-90%), and DEPRESSED C3 levels. Classical clinical presentation includes palpable purpura, arthralgias, and glomerulonephritis. Livedo reticularis, hemorrhagic bullae, acroc yanosis, and urticarial  plaques can also be found. Q/Q(M)-473927 Report a Problem

Patients with Werner �s syndrome typically experience which of the following types of cardiac disease? 1

Hypertrophic cardiomyopathy

2

Aortic aneurysms

3

Premature atherosclerosis

4

Cardiomegaly

5

Mitral valve prolapse 6

Q/Q(M)-479283 Report a Problem Patients with Werner �s syndrome typically experience which of the following types of cardiac disease? 3

Premature atherosclerosis

Werner �s syndrome or Progeria is caused by b y autosomal recessive mutations in WRN (Recql2) gene, which encodes DNA helicase. This defect leads to defects in DNA repair and replication. Patients  prematurely age and essentially experience many diseases of aging early in childhood and teen years. They exhibit tight atrophic skin, relatively large heads for body size, le g ulcers and cannities. Patients experience early, accelerated atherosclerosis atheros clerosis leading to death by myocardial infarction. In addition, type II diabetes, cataracts, osteoarthritis, osteoporosis and hypogonadism are features. Q/Q(M)-479283 Report a Problem

Which of the following is characteristic of diabetic skin? 1

Approximately 20% of diabetics have necrobiosis lipoidica diabeticorum (NLD)

2

The level of cleavage in bullous diabeticorum is subcorneal

3

Candida tropicalis is the most common cause of angular cheilitis

4

There is a well-established association between deep granuloma annulare (GA) and diabetes

5

Yellow skin may occur in up to 10% of diabetics Q/Q(M)-479282 Report a Problem

Which of the following is characteristic of diabetic skin? 5

Yellow skin may occur in up to 10% of diabetics

Perhaps 0.3 to 3% of diabetics have NLD, whereas approximately 20% of NLD patients have diabetes or glucose intolerance. The split in bullous diabeticorum is normally either intraepidermal or subepidermal. C. albicans is the most common cause of diabetes -related yeast infections. The association between GA and diabetes is controversial, but if the two are related, generalized and  perforating GA have been implicated. Yellow skin may occur in up to 10% of diabetic patients and is characterized by diffuse yellow-orange skin. Half of these patients have elevated serum carotene levels. The suggested is cause is elevated consumption of yellow fruits and vegetables in the setting of impaired hepatic metabolism of carotene and subsequent non-enzymatic glycosylation of dermal collagen. Q/Q(M)-479282 Report a Problem

A patient presents with episodic flushing of the face and neck, abdominal pain, wheezing, cough, and diarrhea. Carcinoid syndrome is suspected. Which of the following tests would confirm the diagnosis? 1

Change of urine to green color upon addition of nitrosonaphthol

2

Elevated urinary 5-hydroxyindolacetic acid

3

Elevated 24-hour urine norepinephrine

4

Decreased 24-hour urine vanilylmandelic acid

5

Elevated serum tryptase 7

Q/Q(M)-482359 Report a Problem

A patient presents with episodic flushing of the face and neck, abdominal pain, wheezing, cough, and diarrhea. Carcinoid syndrome is suspected. Which of the following tests would confirm the diagnosis? 2

Elevated urinary 5-hydroxyindolacetic acid

The urine in a patient with carcinoid syndrome features high levels of 5-hydroxyindolacetic acid and changes color to purple upon addition of nitrosonaphthol. Elevated 24-hour urine norepinephrine or vanilylmandelic acid are seen in pheochromocytoma. Mastocytosis features high levels of tryptase. Q/Q(M)-482359 Report a Problem

Which of the following may be an early cutaneous manifestation of Sipple s yndrome (multiple endocrine neoplasia type 2A)? 1

mucosal neuromas

2

Koenen tumor

3

mandibular osteomas

4

multiple pilomatricomas

5

cutaneous amyloidosis Q/Q(M)-482836 Report a Problem

Which of the following may be an early cutaneous manifestation of Sipple s yndrome (multiple endocrine neoplasia type 2A)? 5

cutaneous amyloidosis

Sipple syndrome (multiple endocrine neoplasia type 2A) is an autosomal dominant disorder characterized by the development of pheochromocytomas, medullary thyroid carcinomas, and hyperparathyroidism. The associated mutation is found in the RET proto-oncogene. An early cutaneous manifestation includes the development of lichen or macular am yloidosis (both keratin-derived cutaneous amyloidoses) often localized to interscapular patches, which ma y be pruritic (similar to notalgia paresthetica). The latter arises during teenage years and may be the earliest presenting sign of this disease. Q/Q(M)-482836 Report a Problem The patient demonstrated in figure 6 would have: 1

LDL receptor deficiency

2

Decreased chylomicrons

3

Elevated triglycerides

4

Decreased LDL

5

Decreased triglycerides Q/Q(M)-474939 Report a Problem

8

The patient demonstrated in figure 6 would have: 3

Elevated triglycerides

This patient has eruptive xanthomas, a condition which is classical ly associated with elevated serum triglycerides. Q/Q(M)-474939 Report a Problem

Which of the following is NOT normally associated with dermatomyositis? 1

Ovarian cancer in women

2

Psoriasiform dermatitis of the scalp

3

Elevated aldolase levels

4

Cuticular dystrophy

5

Testicular cancer in men Q/Q(M)-477548 Report a Problem

Which of the following is NOT normally associated with dermatomyositis? 5

Testicular cancer in men

Dermatomyositis is characterized by proximal muscle weakness and a constellation of cutaneous findings. Skin findings include periorbital edema and poikiloderma (heliotrope), Gottron �s sign and  papules, nail fold telangiectasia, cuticular dystrophy, poikiloderma in a shawl distribution and  photosensitivity. Psoriasiform dermatitis and scaling of the palms and soles may occur. Children Children more often display calcinosis cutis and vasculitis. Within the first 3 years of diagnosis there is a higher likelihood of malignancy. Women are at risk for ovarian and breast carcinoma, and men are at risk for gastric carcinoma and lymphoma. Diagnostic findings include elevated creatine kinase (CK) and aldolase, inflammatory muscle biopsy changes or abnormal muscle MRI, and abnormal EMG. 9

Treatment includes corticosteroids, methotrexate and other immunosuppressive medications. Q/Q(M)-477548 Report a Problem Which of the following is TRUE about diabetic-related skin disease? 1

Bullae are common on the thighs

2

Yellow skin affects the majority of diabetic patients

3

Diabetic dermopathy affects the upper back

4

Necrobiosis lipoidica affects approximately 20% of diabetics

5

Direct immunofluorenscence is usually negative in bullous diabeticorum Q/Q(M)-477563 Report a Problem

Which of the following is TRUE about diabetic-related skin disease? 5

Direct immunofluorenscence is usually negative in bullous diabeticorum

Approximately 30% of diabetic patients have cutaneous manifestations of their disease. Diabetic dermopathy, or �shin spots� is the most common cutaneous association with diabetes. Patients are generally long-standing diabetics, and are associated with symmetrical brownish, atrophic plaques on the shins. Acanthosis nigricans is more common in black and Hispanic dia betics, and may be a component of the HAIRAN (hyperandrogen, insulin resistance, a canthosis nigricans) syndrome. Hyperpigmented velvety plaques are most often found in the flexures. Waxy, thick skin and limited  joint mobility are both related to poor glucose glucose control. Scleredema diabeticorum, which most often causes thickened plaques of the upper back and neck most often affects type II diabetics. Necrobiosis lipoidica diabeticorum (NLD) is present in only 0.3 to 3% of diabetics, but perhaps 20% of patients with NLD have diabetes or glucose intolerance. Sharply-demarcated yellow-brown, telangiectatic  plaques are found on the tibial surfaces. Ulceration may occur. Yellow skin may affect approximately 10% of diabetics and presents diffusely. Serum carotene levels may be elevated. Perforating disorders may be related, especially in the setting setti ng of renal disease. In the setting sett ing of hypertriglyceridemia, eruptive xanthomata may be present. The association between diabetes and granuloma annulare (GA) is controversial. If truly an association, the generalized and perforating forms of GA are most commonly associated. Infectious associations include candidiasi s, which may present an angular cheilitis, chronic  paronychia, interdigital or intertriginous disease. Cutaneous bacterial infections, including group A and B streptococci, pseudomonas aerugenosa and corynebacterium are more common. Dermatophyte infections are not more common, although tinea pedis may increase the risk for cellulitis in diabetic  patients. Finally, rhinocerebral mucormycosis is a potential disasterous complication of uncontrolled hyperglycemia with ketoacidosis. Mortality rates can approach 35%. Q/Q(M)-477563 Report a Problem Which of the following is FALSE regarding Fabry �s disease? 1

It is inherited in an X-linked recessive fashion

2

It is associated with acral parasthesias

3

It is inherited in an X-linked dominant fashion

4

It may be associated with renal failure

5

The etiology is a defect in alpha-galactosidase A Q/Q(M)-477562 Report a Problem 10

Which of the following is FALSE regarding Fabry �s disease? 3

It is inherited in an X-linked dominant fashion

Fabry�s disease is an X-linked lysosomal disorder that leads to excessive deposition of neutral glycosphingolipids in the vascular endothelium. The disorder is caused by a deficiency of alphagalactosidase-A leading to progressive endothelial accumulation of glycosphingolipids. This accumulation accounts for the associated clinical abnormalities of skin, eye, kidney, heart, brain, and  peripheral nervous system. Acroparesthesias are a frequent presenting symptom of Fabry�s disease. The primary cutaneous manifestation are angiokeratomata (angiokeratoma corporis diff usum), which are most common in a � bathing-trunk � distribution. Lens opacities and retinal and conjunctival vascular malformations may be found in the eyes. Patients may experience cardiac disease, stroke or renal failure. Q/Q(M)-477562 Report a Problem

A middle-aged gentleman who avoided healthcare dies suddenly from gastrointestinal hemorrhage. Post-mortem examination reveals multiple soft blue compressible tumors on the trunk, arms, and tongue. Blue rubber bleb nevus syndrome is caused by a mutation in what gene? 1

VMCM1

2

ENG

3

VEGF

4

PTEN

5

RET Q/Q(M)-482738 Report a Problem

A middle-aged gentleman who avoided healthcare dies suddenly from gastrointestinal hemorrhage. Post-mortem examination reveals multiple soft blue compressible tumors on the trunk, arms, and tongue. Blue rubber bleb nevus syndrome is caused by a mutation in what gene? 1

VMCM1

Blue rubber bleb nevus syndrome is a rare sporadic or autosomal dominant disorder with soft compressible blue tumors on the trunk and arms. Nocturnal pain is characte ristic. Gastrointestinal hemangiomas can cause hemorrhage. Mutations in the VMCM1 gene are reported. Q/Q(M)-482738 Report a Problem

Which of the following is not a feature of Cronkhite-Canada syndrome? 1

Diarrhea

2

Alopecia

3

Lung carcinoma

4

Dystrophic nails

5

Hyperpigmented macules Q/Q(M)-477382 Report a Problem 11

Which of the following is not a feature of Cronkhite-Canada syndrome? 3

Lung carcinoma

Cronkhite-Canada syndrome is a rare, non-familial disease ch aracterized by patchy alopecia, nail changes or loss, lentigines, inflammatory polyps, abdominal pain, and a protein losing enteropathy. Q/Q(M)-477382 Report a Problem Which of the following diseases is caused by an enzymatic defect that occurs in the mitochondria? 1

Porphyria Cutanea Tarda

2

Acute Intermittent Porphyria

3

Congenital Erythropoeitic Porphyria

4

Erythropoeitic Protoporphyria

5

Hepatoerythropoetic Porphyria Q/Q(M)-476508 Report a Problem

Which of the following diseases is caused by an enzymatic defect that occurs in the mitochondria? 4

Erythropoeitic Protoporphyria

Defects in many of the enzymes involved in heme synthesis are responsible for porphyrias. The first and last three steps of the heme synthesis pathway occur in the mitochondria. Q/Q(M)-476508 Report a Problem Features of Cushing disease include all of the following EXCEPT? 1

Facial plethora

2

Striae

3

Hypertension

4

Suppression of corticotropin occurs with administration of dexamethasone

5

Hirsutism Q/Q(M)-477550 Report a Problem

Features of Cushing disease include all of the following EXCEPT? 4

Suppression of corticotropin occurs with administration of dexamethasone

Cushing syndrome is caused by excess levels of either exogenously administrated glucocorticoids or endogenous overproduction of cortisol. The syndrome is most commonly caused by the therapeutic administration of exogenous glucocorticoids. The term "Cushing's disease" is reserved for Cushing's syndrome that is caused by excessive secretion of adrenocorticotropin hormone (ACTH) by a pituitary tumor, usually an adenoma. Other endogenous sources include an adrenal tumor and other malignancies (i.e. oat cell carcinoma of the lung). Patients develop moon facies, facial plethora, supraclavicular fat pads, buffalo hump, truncal obesity, and striae. In addition, proximal muscle weakness, easy bruising, weight gain, hirsutism, and, in children, growth retardation may occur. Systemic sequelae include hypertension, osteopenia, diabetes mellitus, and impaired immune function. Excess endogenous or exogenous glucocorticoids can result in a leukocytosis and hypokalemic 12

metabolic acidosis. Urine cortisol levels 3 to 4 times normal are diagnostic. The dexamethasone suppression test is a valuable diagnostic tool. For this study, 1mg is given at 11pm and the 8am cortisol level is tested. A level of >10 �g/dL is diagnostic. In patients with Cushing �s disease, the dexamethasone does not suppress corticotropin. Corticotropin-releasing hormone (CRH) stimulation test is also used to distinguish patients with pituitary adenomas and those with ectopic ACTH syndrome or cortisol-secreting adrenal tumors. Q/Q(M)-477550 Report a Problem

All of the following are true regarding the condition pictured EXCEPT: 1

Men with this condition may be at increased risk for lymphoma

2

Associated calcinosis cutis may be seen in pediatric patients

3

Age-appropriate cancer screening is recommended

4

May be associated with a psoriasiform scalp dermatitis

5

Elevated aldolase is more specific than CPK Q/Q(M)-474078 Report a Problem

All of the following are true regarding the condition pictured EXCEPT: 5

Elevated aldolase is more specific than CPK

All of the statements regarding dermatomyositis are a re true except for statement E. Elevated muscle enzymes are found in dermatomyositis, but elevated CPK is more specif ic than aldolase. Diagnosis is also established with muscle biopsy showing evidence of inflammation and abnormal EMG. Q/Q(M)-474078 Report a Problem

In patients with mixed cryoglobulinema associated with hepatitis C, the most likely laboratory abnormality is: 1

Elevated rheumatoid factor

2

+ ANA

3

+ p-ANCA

4

Decreased cryoglobulins 13

5

Elevated hematocrit Q/Q(M)-474549 Report a Problem

In patients with mixed cryoglobulinema associated with hepatitis C, the most likely laboratory abnormality is: 1

Elevated rheumatoid factor

In patients with mixed cryoglobulinemia, the most likely laboratory abnormality among the options listed is an elevated rheumatoid factor. Q/Q(M)-474549 Report a Problem

Which of the following is NOT true regarding calciphylaxis? 1

Patients with proximally-located lesions have a better prognosis than those with acral lesions

2

May be treated with parathyroidectomy

3

May be present with retiform purpura

4

Histologic findings include medial calcification and intimal hyperplasia of small arteries and arterioles

5

Has an associated mortality of 60-80% Q/Q(M)-473946 Report a Problem

Which of the following is NOT true regarding calciphylaxis? 1

Patients with proximally-located lesions have a better prognosis than those with acral lesions

Distribution of lesions in calciphylaxis may predict prognosis; those with acral lesions have a better outcome than those with proximally located lesions. Q/Q(M)-473946 Report a Problem Which of the following laboratory abnormalities is most common in patients with cholest erol emboli? 1

Neutrophilia

2

Elevated amylase

3

Hypercalcemia

4

Eosinophilia

5

Anemia Q/Q(M)-482533 Report a Problem

Which of the following laboratory abnormalities is most common in patients with cholest erol emboli? 4

Eosinophilia

Eosinophilia develops within 3 days of embolization in 70-80% of patients and may remain elevated for up to 1 month. Patients with cholesterol emboli typically have an elevated ESR and C-reactive 14

 protein as well. Leukocytosis can also be seen in up to half of patients. Q/Q(M)-482533 Report a Problem

Which of the following is NOT associated with Cronkhite-Canada syndrome? 1

Lentigines

2

Adenomatous gastrointestinal polyps

3

Fibrocystic breast disease

4

Onycholysis

5

Weight loss Q/Q(M)-477546 Report a Problem

Which of the following is NOT associated with Cronkhite-Canada syndrome? 3

Fibrocystic breast disease

Cronkhite-Canada syndrome (CCS) is a rare, sporadically occurring, non-inherited disorder characterized by generalized gastrointestinal polyps, pol yps, cutaneous pigmentation, alopecia, and onychodystrophy. onychodystrophy. The pathogenesis is unknown. The GI polyps are hamartomatous and mali gnant degeneration is unusual. Cutaneous signs include circumscribed lentiginous hyperpigmentation, alopecia and nail dystrophy. Often, symptoms appear in the sequence of gastrointestinal symptoms, weight loss, weakness, edema, and then cutaneous changes after weeks or months. CCS is a progressive disease, and often carriers a poor prognosis primarily because of difficulties with fluid and electrolyte management. Fibrocystic breast disease is associated as sociated with Cowden?s disease (multiple hamartoma syndrome). Q/Q(M)-477546 Report a Problem Which of the following cutaneous findings are characteristic for Vohwinkles S yndrome? 1

Honeycombed diffuse palmoplantar keratoderma, pseudoainhum with autoamputation, starshaped keratosis over knuckles, nail dystrophy, and alopecia

2

Dense depigmented lusterless hair, pili torti, doughy skin, diffuse cutaneous hypopigmentation

3

Transient erythroderma at birth, palmoplantar keratoderma, follicular hyperkeratosis, sc arring alopecia, dystrophic nails

4

Rapidly progressive alopecia of all hair-bearing ar eas, onycholysis, onychoschizia, onychomadesis, hyperpigmented macules on extremities

5

Oral papillomatosis, palmoplantar keratoses, acral keratoses, lipomas, hemangiomas, scrotal tongue

Q/Q(M)-473950 Report a Problem Which of the following cutaneous findings are characteristic for Vohwinkles S yndrome? 1

Honeycombed diffuse palmoplantar keratoderma, pseudoainhum with autoamputation, star shaped keratosis over knuckles, nail dystrophy, and alopecia

Vohwinkles Syndrome is an autosomal dominant disorder caused by a mutation of the GJB2 gene which encodes Connexin 26. Clinical features include deafness, as well as the cutaneous findings described in choice A. Q/Q(M)-473950 Report a Problem

15

Which of the following is true regarding treatment of inflammatory dermatoses with potassium iodide? 1

The Wolff-Chaikoff effect must be considered

2

Binding of excess organic iodide in the thryoid gland may occur

3

Thyroid hormone synthesis may be inhibited

4

None of the answers are correct

5

All of these answers are correct Q/Q(M)-473944 Report a Problem

Which of the following is true regarding treatment of inflammatory dermatoses with potassium iodide? 5

All of these answers are correct

The Wolff-Chaikoff effect is described as the th e binding of excess organic iodide in the thyroid gland with resultant inhibition of thyroid hormone synthesis. This can occur in the setting of patients with erythema nodosum (or other inflammatory dermatoses) being treated with potassium i odide. Q/Q(M)-473944 Report a Problem

Regarding eruptive xanthomata, which of the following is TRUE? 1

They are associated with type II and III hyperlipidemias

2

They are associated with calcium channel blockers

3

There is no association with ethanol consumption

4

They are associated with type I, IV, and V hyperlipidemias

5

They favor the flexor surfaces of the extremities Q/Q(M)-477558 Report a Problem

Regarding eruptive xanthomata, which of the following is TRUE? 4

They are associated with type I, IV, and V hyperlipidemias

Eruptive xanthomata appear as erythematous to yellow papule, from1 to 4mm in diameter, generally distributed over the extensor arms, hands, and buttocks. They are present in response to either primary or secondary hypertriglyceridemia. Primary causes are dis cussed below. Secondary causes include obesity, diabetes, excessive alcohol consumption, estrogens, and systemic retinoids. Type I hyperlipidemia is caused by lipoprotein lipase deficiency and patients have elevated levels of triglycerides (TG) and chylomicrons (chylo). Type I may be associated with eruptive xanthomata, lipemia retinalis, abdominal pain, pancreatitis, and hepatosplenomegaly. Type IIA is heteroz ygous for apolipoprotein B deficiency, and results in increased low-densit y lipoproteins (LDL). Patients may show tendinous or tuberous xanthomata and are at high risk for coronary artery disease ( CAD) and stroke. Type IIB is homozygous for apolipoprotein B deficiency and patients have elevated LDL, very low-density lipoprotein (VLDL) and TG. Intertriginous a nd tuberous xanthomata in addition to advanced atherosclerosis, CAD and stroke at an earl y age are associated with type t ype IIB. Type III is caused by apolipoprotein E deficiency resulting in elevated intermediate-density lipoproteins (IDL) and TG, and displays palmar/plantar, tendinous and tuberous xanthomata. This disease is associated with diabetes, gout, CAD and stroke. Type IV disease is characteriz ed by increased TG and VLDL and 16

 patients may have eruptive xanthomata along with CAD, diabetes and stroke. Type V is caused by an apolipoprotein C2 defect resulting in increased TG, chylo, and VLDL. Eruptive xanthomata, diabetes, hepatosplenomegaly, lipemia retinalis and pancreatitis are associated with type V. Q/Q(M)-477558 Report a Problem A 58 year old female with lifelong Type 1 diabetes mellit us has end-stage renal failure. What is the most common cutaneous manifestation of end stage renal disease? 1

Pruritus

2

Nephrogenic systemic fibrosis

3

Hyperpigmentation

4

Acquired perforating dermatosis

5

Skin pallor Q/Q(M)-482735 Report a Problem

A 58 year old female with lifelong Type 1 diabetes mellit us has end-stage renal failure. What is the most common cutaneous manifestation of end stage renal disease? 1

Pruritus

There are may cutaneous manifestations of end-stage end-st age renal disease. Chronic anemia can cause skin  pallor. Deposition of carotenoids can give the skin a yellowish yellowish hue. Photo-distributed hyperpigmentation and ecchymoses are also commonly c ommonly seen. The most common cutaneous manifestation, however, is pruritus. Other cutaneous findings include metastatic calcification, acquired  perforating dermatosis, and nephrogenic systemic fibrosis. Q/Q(M)-482735 Report a Problem A patient with congenital hypertrophy of retinal epithelium is most likely to have: 1

An autosomal dominant mutation in the MSH2 gene

2

Pheochromocytoma

3

Adenomatous polyposis

4

Tram-track calcifications on head radiograph

5

Peg-shaped teeth

Q/Q(M)-477553 Report a Problem A patient with congenital hypertrophy of retinal epithelium is most likely to have: 3

Adenomatous polyposis

Congenital hyperpigmentation of the retinal pigment (CHRPE) is an early feature of Gardner syndrome (GS). It is found in approximately 60% of patients with GS. GS is an autosomal dominant disorder characterized by precancerous intestinal polyposis and subsequent adenocarcinoma of the gastrointestinal tract. Cutaneous manifestations include epidermoid cysts, osteomas, desmoids and fibrous tumors. A mutation in the adenomatous polyposis coli (APC) gene, a tumor suppressor gene, is responsible for the disease. Most patients develop colon carcinoma by the 2nd or 3rd decade. Therefore, prophylactic colectomy is warranted. Mutations in the MSH2 gene are found in Muir-Torre syndrome. Pheochromocytomas Pheochromocytomas are found in multiple endocrine neoplasia (MEN) s yndromes IIa and 17

IIb. Tram track calcifications are found in Sturge-Weber syndrome. Peg-shaped teeth are found in multiple syndromes including ectodermal dysplasia. Q/Q(M)-477553 Report a Problem What is the most common malignancy associated with t his condition at this location? 1

Breast cancer

2

Gastric cancer

3

Thyroid cancer

4

Pancreatic cancer

5

Melanoma Q/Q(M)-476532 Report a Problem

What is the most common malignancy associated with this condition at this location? 2

Gastric cancer

Malignant acanthosis nigricans usually presents with sudden onset and is rapidly progressive. It may be associated with diffuse keratodermas of the palms and soles or eruptive seborrheic keratoses. Q/Q(M)-476532 Report a Problem

An end stage AIDS patient with tuberculosis presents with diffuse hyperpigmentation of both sunexposed and unexposed areas. The palmer creases are ma rkedly hyperpigmented and the patient is very ill. The most likely diagnosis is: 1

Argyria

2

Lymphoma

3

Tinea versicolor 18

4

Kaposi's sarcoma

5

Addison disease Q/Q(M)-479613 Report a Problem

An end stage AIDS patient with tuberculosis presents with diffuse hyperpigmentation of both sunexposed and unexposed areas. The palmer creases are ma rkedly hyperpigmented and the patient is very ill. The most likely diagnosis is: 5

Addison disease

Addison disease is caused by destruction of the adrenal glands by any cause. Tuberculosis used to be the primary cause, now the most common cause is auto-immune destruction. Other causes of adrenal gland destruction include; coccidiomycosis, cryptococcosis, histoplasmosis, sarcoidosis, metastatic tumor and amyloidosis. Q/Q(M)-479613 Report a Problem

A patient presents with plane/palmar xanthomas. The most likel y genetic disorder would be: 1

familial lipoprotein lipase deficiency

2

Familial hypertriglyceridemia

3

Familial hypercholesterolemia

4

Familial dysbetalipoproteinemia

5

Cerebrotendinous xantomatosis Q/Q(M)-482319 Report a Problem

A patient presents with plane/palmar xanthomas. The most likel y genetic disorder would be: 4

Familial dysbetalipoproteinemia

Patients with Type III hyperlipidemia have both elevated triglyceride levels and cholesterol levels in the plasma. A genetic basis for the primary disorder, familial dysbetalipoproteinemia, has been well established. These patients present as adults with premature atherosclerosis and a nd xanthomas, particularly  plane (palmar) xanthomas. Q/Q(M)-482319 Report a Problem

A patient with gastric cancer develops acanthosis nigricans and a sudden eruption of numerous warty stuck-on papules on the trunk. What other finding may be seen? 1

Numerous wart-like lesions on the dorsal hands and wrists

2

Follicular spicules on the nose

3

Carpal tunnel syndrome

4

Periorbital pupura

5

Migratory thrombophlebitis Q/Q(M)-482354 Report a Problem 19

A patient with gastric cancer develops acanthosis nigricans and a sudden eruption of numerous warty stuck-on papules on the trunk. What other finding may be seen? 1

Numerous wart-like lesions on the dorsal hands and wrists

Florid cutaneous papillomatosis is the rapid onset of numerous wart-like lesions on the backs of the hands and wrists that is associated with pruritus, acanthosis nigricans, and the sign of Leser-Trelat. All reported cases are associated with internal malignancy, most commonly gastric carcinoma. Follicular  papules on the nose are seen in multiple myeloma. Carpal tunnel tunnel syndrome and periorbital purpura can  be seen in primary systemic amyloidosis with multiple myeloma. migratory migratory thrombophlebitis is associated with pancreatic cancer. Q/Q(M)-482354 Report a Problem

A 12 year old boy is referred for evaluation of a r ash in his groin. It has become progressively worse over the preceding three years. Discrete erythematous papules are seen on the lower anterior abdomen, upper thighs, and buttocks. Biopsy of a representative papule reveals an angiokeratoma. An evaluation  by a nephrologist reveals proteinura. A diagnosis diagnosis of Fabry\'s disease is made. Renal failure in this syndrome is related to accumulation of what substance in the kidney? 1

Ceramide trihexoside

2

Galabiosylceramide

3

Globotriaosylceramide

4

Mucopolysaccharide

5

Alpha-galactosidase Q/Q(M)-482734 Report a Problem

A 12 year old boy is referred for evaluation of a r ash in his groin. It has become progressively worse over the preceding three years. Discrete erythematous papules are seen on the lower anterior abdomen, upper thighs, and buttocks. Biopsy of a representative papule reveals an angiokeratoma. An evaluation  by a nephrologist reveals proteinura. A diagnosis diagnosis of Fabry\'s disease is made. Renal failure in this syndrome is related to accumulation of what substance in the kidney? 1

Ceramide trihexoside

Fabry\'s disease is an x-linked recessive disorder characterized by angiokeratomas in a bathing trunk distribution, hypohidrosis, corneal opacities, and acral paresthesias. The diagnosis is confirmed by decreased levels of alph-galactosidase in white blood cells, serum, and fibroblasts. Renal fail ure is due to accumulation of ceramide trihexoside. Q/Q(M)-482734 Report a Problem

All of the following are true regarding Henoch-Schonlein Purpura EXCEPT: 1

It is an IgG mediated small vessel vasculitis

2

Direct immunofluorescence of lesional and peri-lesional skin will demonstrate C3 and fibrin deposits in small vessel wall

3

It is often preceded by an upper respiratory infection

4

It may be complicated by intussusception

5

It is self-resolving 20

Q/Q(M)-473963 Report a Problem

All of the following are true regarding Henoch-Schonlein Purpura EXCEPT: 1

It is an IgG mediated small vessel vasculitis

Henoch-Schonlein Purpura (HSP) is an IgA mediated small vessel vasculitis. Clinically, patients  present with palpable purpura of the lower extremities and buttocks, GI vasculitis, and glomerulonephritis. Q/Q(M)-473963 Report a Problem

Which of the following porphyria cutanea tarda associations has a direct relationship to the level of urine uroporphyrins? 1

Dystrophic calcifications

2

Estrogen levels

3

Sclerodermoid changes

4

RBC fluorescence

5

Hypertrichosis Q/Q(M)-482479 Report a Problem

Which of the following porphyria cutanea tarda associations has a direct relationship to the level of urine uroporphyrins? 3

Sclerodermoid changes

Connective tissue dystrophic calcifications, increased estrogen levels, hypertrichosis and sclerodermoid changes are all associated with porphyria cutanea tarda, but sclerodermoid changes are the only finding with a direct relationship to urine uroporphyrin levels. Urine fluorescence occurs in PCT, not RBC fluorescence. Q/Q(M)-482479 Report a Problem A patient develops cold-exacerbated dusky acral plaques consistent with pernio. The systemic condition most likely to manifest such lesions is: 1

Lupus Erythematosus

2

Dermatomyositis

3

Acquired Immunodeficiency Syndrome

4

Hepatitis C

5

Diabetes Mellitus Q/Q(M)-482217 Report a Problem

A patient develops cold-exacerbated dusky acral plaques consistent with pernio. The systemic condition most likely to manifest such lesions is: 21

1

Lupus Erythematosus

Chilblain lupus is a rare manifestation of lupus l upus erythematosus. Lesions resemble common pernio clinically, but should have histological findings consistent with lupus, such as interface changes. These lesions may not resolve upon warming and often respond poorly to other lupus treatments such as antimalarials. Q/Q(M)-482217 Report a Problem

This autosomal dominant condition is characterized by trichodiscomas, fibroepithelial polyps, and fibrofolliculomas: 1

POEMS syndrome

2

Rombo syndrome

3

Birt-Hogg-Dube syndrome

4

Proteus syndrome

5

Cowden's disease Q/Q(M)-474550 Report a Problem

This autosomal dominant condition is characterized by trichodiscomas, fibroepithelial polyps, and fibrofolliculomas: 3

Birt-Hogg-Dube syndrome

Birt-Hogg-Dube is an autosomal dominant condition characterized by trichodiscomas, fibroepilethial  polyps and fibrofolliculomas. Q/Q(M)-474550 Report a Problem

All of the following disorders have an increased risk of systemic malignancy except 1

Rothmund Thompson syndrome

2

Cockayne syndrome

3

Bloom syndrome

4

Werner syndrome

5

Xeroderma pigmentosum Q/Q(M)-482318 Report a Problem

All of the following disorders have an increased risk of systemic malignancy except 2

Cockayne syndrome

Patients with Rothmund thompson have early photosensitivit y and poikilodermatous skin changes,  juvenile cataracts, skeletal dysplasias, and a predisposition to osteosarcoma and skin cancer. Patients with Bloom syndrome have an increased risk for GI malignancies and l ymphoma. Patients with xeroderma pigmentosum can develop ocular melanoma. Patients with Werner syndrome can develop thyroid and hematologic malignancies, sarcomas, and meningiomas. Cockayne syndrome patients have  photosensitivity, short stature, premature aging, visual problems, neurologic neurologic deficits, but no internal malignancies. 22

Q/Q(M)-482318 Report a Problem

Which step is rate-limiting in the synthesis of protoporphyrinogen? 1

Aminolevulinic acid synthase

2

Aminolevulinic acid dehydratase

3

Porphobillinogen deaminase

4

Coproporphyrinogen Coproporphyrinogen oxidase

5

Ferrochelatase Q/Q(M)-482393 Report a Problem

Which step is rate-limiting in the synthesis of protoporphyrinogen? 1

Aminolevulinic acid synthase

The rate-limiting step in the heme synthesis pathway is the synthesis of delta aminolevulanic acid from glycine and succinyl CoA via aminolevulinic acid synthase. This step takes place i n the mitochondria. Porphobillinogen deaminase is defective in acute intermittent porphyria, coproporphyrinogen coproporphyrinogen oxidase in in hereditary coproporphyria and protoporphyrinogen oxidase in variegate porphyria. Q/Q(M)-482393 Report a Problem Which of the following is true regarding nephrogenic fibrosing dermopathy? 1

Has a rapidly progressive but reversible course

2

Is associated with a paraproteinemia

3

Is associated with peripheral eosinophilia

4

May be associated with antiphospholipid antibodies

5

Is associated with a dramatic increase in dermal mucin Q/Q(M)-473947 Report a Problem

Which of the following is true regarding nephrogenic fibrosing dermopathy? 4

May be associated with antiphospholipid antibodies

 Nephrogenic fibrosing dermopathy has an indolent course and treatment is usually not satisfactory. It is not associated with a paraproteinemia or peripher al eosinophilia. Some patients have been reported to have antiphospholipid antibodies. Histopathology demonstrates a minimal to slight increase in dermal mucin. Q/Q(M)-473947 Report a Problem All of the following are seen more commonly in Crohn's disease than in ulcerative colitis EXCEPT: 1

Oral cobblestoning

2

Polyarteritis nodosa 23

3

Pyostomatitis vegetans

4

Perineal fistulas

5

Perineal fissures Q/Q(M)-473968 Report a Problem

All of the following are seen more commonly in Crohn's disease than in ulcerative colitis EXCEPT: 3

Pyostomatitis vegetans

Pyoderma vegetans consists of vegetating plaques and vesicopustules of intertriginous areas than heal with hyperpigmentation. When the process involves mucosal surfaces it is called pyostomatitis vegetans. These processes are associated with ulcerative colitis, not Crohn's disease. Q/Q(M)-473968 Report a Problem

A 7 year old boy is seen for "acne". Hypopigmented H ypopigmented patches are seen on the trunk and flesh-colored  papules are seen around the nails. Biopsy of one one of the papules of the face reveals an angiofibroma. A review of his chart reveals that he takes medications for seizures. What is the most common neoplasm associated with the most likely disorder? 1

Renal angiomyolipoma

2

Renal cell carcinoma

3

Gastric carcinoma

4

Transitional cell carcinoma

5

Prostate carcinoma Q/Q(M)-482733 Report a Problem

A 7 year old boy is seen for "acne". Hypopigmented H ypopigmented patches are seen on the trunk and flesh-colored  papules are seen around the nails. Biopsy of one one of the papules of the face reveals an angiofibroma. A review of his chart reveals that he takes medications for seizures. What is the most common neoplasm associated with the most likely disorder? 1

Renal angiomyolipoma

Tuberous sclerosis is an autosomal dominant disorder with seizures, mental retardation, and characteristic skin findings, including hypopigmented macules, facial angiofibromas, periungual fibromas, and collagenomas. The most common associated tumor is renal angiomyolipoma. Rarely, renal cell carcinoma can develop. Q/Q(M)-482733 Report a Problem What is the most common primary site of a carcinoid tumor? 1

Liver

2

Stomach

3

Appendix

4

Duodenum 24

5

Ileum Q/Q(M)-482358 Report a Problem

What is the most common primary site of a carcinoid tumor? 3

Appendix

The most common site of carcinoid tumor is the appendix. Clinical symptoms of episodic flushing, abdominal pain, diarrhea, wheezing, and a pellagra-like dermatosis (due to s hunting of tryptophan to serotonin) etc present with metastases to the liver. The most common original source of metastases is the ileum. Q/Q(M)-482358 Report a Problem A 70-year old male develops hyperpigmented velvety plaques on his lips, dorsal hands and feet, and in his axilla. What underlying malignancy is most commonly associated with this finding? 1

Gastric Carcinoma

2

Lung Carcinoma

3

Ovarian Carcinoma

4

Lymphoma

5

Breast Carcinoma Q/Q(M)-482238 Report a Problem

A 70-year old male develops hyperpigmented velvety plaques on his lips, dorsal hands and feet, and in his axilla. What underlying malignancy is most commonly associated with this finding? 1

Gastric Carcinoma

Acanthosis Nigricans can be associated with obesity, insulin resistance, Crouzonâ ™s syndrome, congenital lipodystrophy, and internal malignancy. 90% of related malignancies are tumors within the abdominal cavity. Adenocarcinoma of the stomach is the most common. Malignancy related acanthosis nigricans often occurs in the setting of weight wei ght loss, helping distinguish it from other associated diseases. Q/Q(M)-482238 Report a Problem

All of the following are true regarding the condition pictured EXCEPT: Perilesional direct immunofluroescence shows granular IgA in the dermal papillae and at the 1 dermoepidermal junction 2

Only 20% of patients have a gluten-sensitive enteropathy

3

It is associated with HLA-DQ2, HLA-DR3, and HLA-B8

4

It is associated with Hashimoto's thyroiditis

5

Cutaneous findings are due to autoantibodies to epidermal transglutaminase Q/Q(M)-474077 Report a Problem

25

All of the following are true regarding the condition pictured EXCEPT: 2

Only 20% of patients have a gluten-sensitive enteropathy

All of the statements regarding dermatitis herpetiforms, or "Duhring's Disease," are true except for statement B. Virtually all DH patients have gluten-sensitive enteropathy, although only 20% of them have symptoms (such as diarrhea, steatorrhea, weight loss, bloating, and malabsorption). Q/Q(M)-474077 Report a Problem

A 6 month-old has a verrucous plaque on the mucosal surface of the lower lip. Skin biopsy is consistent with Riga-Fede disease. You should refer the patient to: 1

An ophthalmologist

2

A gastroenterologist

3

A neurologist

4

A hematologist

5

An otolarngologist Q/Q(M)-477210 Report a Problem

A 6 month-old has a verrucous plaque on the mucosal surface of the lower lip. Skin biopsy is consistent with Riga-Fede disease. You should refer the patient to: 3

A neurologist

Riga-Fede disease is a benign ulcerative granulomatous process that occurs in reaction to chronic, repetitive trauma of the oral mucosa by the teeth. Clinically, it appears as firm, verrucous plaques. It may be associated with an underlying developmental anomaly or underlying neurologic disorder. Q/Q(M)-477210 Report a Problem

A 64-year old man develops yellowish periorbital plaques that occasionally ulcerate and heal with scarring. What is the most likely associated lab finding? 1

Monoclonal gammopathy 26

2

Positive antinuclear antibodies

3

Elevated creatinine

4

Elevated triglycerides

5

Increased thyroid stimulating hormone Q/Q(M)-482242 Report a Problem

A 64-year old man develops yellowish periorbital plaques that occasionally ulcerate and heal with scarring. What is the most likely associated lab finding? 1

Monoclonal gammopathy

The patient has necrobiotic xanthogranuloma. Lesions develop periorbitally and in flexural areas, often extend deeply into the dermis or subcutis, and heal with scarring. These lesions are typically associated with a monoclonal gammopathy, usually IgG. Patients with necrobiotic xanthogranuloma rarely develop myeloma. Leukopenia and hepatosplenomegaly are also often seen. Q/Q(M)-482242 Report a Problem

Hyperkeratotic follicular nasal papules have been described as a paraneoplastic sign in the setting sett ing of which neoplasm? 1

Multiple myeloma

2

Castleman's tumor

3

AML

4

Adenocarcinoma of the lung

5

Renal cell carcinoma Q/Q(M)-477434 Report a Problem

Hyperkeratotic follicular nasal papules have been described as a paraneoplastic sign in the setting sett ing of which neoplasm? 1

Multiple myeloma

Hyperkeratotic follicular nasal papules have been described as a paraneoplastic phenomenon in the setting of multiple myeloma. AML is associated with Sweet's syndrome and Castleman's tumor is associated with paraneoplastic pemphigus. Q/Q(M)-477434 Report a Problem

Which of the following statements regarding multiple endocrine neoplasia s yndromes is true? 1

MEN Type IIB is also known as Sipple's Syndrome

2

MEN Type IIA is also known as Wermer's Syndrome

3

Multiple mucosal neuromas are seen in association with MEN Type IIA

4

Lichen or macular amyloidosis is seen in association with MEN Type IIA

5

Patients with MEN Type IIB are at increased risk for developing follicular thyroid carcinoma Q/Q(M)-473932 Report a Problem 27

Which of the following statements regarding multiple endocrine neoplasia s yndromes is true? 4

Lichen or macular amyloidosis is seen in association with MEN Type IIA

MEN Type I is also known as Wermer's Syndrome. MEN Type IIA is also known as Sipple's Syndrome. Mucosal neuromas are seen in association with MEN Type IIB. Statement D i s correct. Patients with MEN Type IIB are at increased risk ri sk for developing medullary thyroid carcinoma. Q/Q(M)-473932 Report a Problem

All of the following statements regarding cryoglobulinemia are true EXCEPT: 1

Type I is composed of monoclonal IgG and polyclonal IgM

2

Type II is composed of polyclonal IgG and monoclonal IgM

3

Type III is composed of polyclonal IgG and polyclonal IgM

4

80% of cases of mixed cryoglobulinemia are associated with Hepatitis C infection

5

None of these answers are correct (all statements are true) Q/Q(M)-473925 Report a Problem

All of the following statements regarding cryoglobulinemia are true EXCEPT: 1

Type I is composed of monoclonal IgG and polyclonal IgM

Type I is composed of monoclonal m onoclonal immunoglobulins. Q/Q(M)-473925 Report a Problem

Which of the following is a paraneoplastic disease most often associated with lung carcinoma? 1

Hypertrichosis lanuginosa acquisita

2

Dermatomyositis

3

Acanthosis nigricans

4

Paraneoplastic pemphigus

5

Erythroderma Q/Q(M)-473955 Report a Problem

Which of the following is a paraneoplastic disease most often associated with lung carcinoma? 1

Hypertrichosis lanuginosa acquisita

Hypertrichosis lanuginosa acquisita is the abrupt onset of downy, soft, non-pigmented hair of the face, trunk, and extremities. It may have an associated gloss itis. It is associated wi th underlying lung carcinoma and may resolve with treatment of the underlying malignancy. Q/Q(M)-473955 Report a Problem Patients with this syndrome are at increased risk r isk for developing Lhermite-Duclos disease: 1

Bourneville's Disease 28

2

Nail-Patella Syndrome

3

MEN Type IIA

4

Fabry's Disease

5

Cowden's Syndrome Q/Q(M)-473934 Report a Problem

Patients with this syndrome are at increased risk r isk for developing Lhermite-Duclos disease: 5

Cowden's Syndrome

Patients with Cowden's Syndrome (multiple hamartoma syndrome) are at increased risk for Lhermite Duclos disease (dysplastic gangliocytoma of the cerebellum). Q/Q(M)-473934 Report a Problem Regarding paraneoplastic pemphigus, which of the following is TRUE? 1

Desmoplakin, one of the molecular antigens, has a molecular weight of 190kd

2

Metastatic squamous cell carcinoma of the skin is a common cause

3

Granular C3 deposition is found at the dermoepidermal junction on direct immunofluorescence

4

Monkey esophagus is the preferred substrate for indirect immunofluorescence

5

Non-Hodgkins lymphoma is rarely associated Q/Q(M)-477547 Report a Problem

Regarding paraneoplastic pemphigus, which of the following is TRUE? 3

Granular C3 deposition is found at the dermoepidermal junction on direct immunofluorescence

Paraneoplastic pemphigus (PNP) is characterized by painful stomatitis, which is extremely resistant to therapy. Cutaneous lesions are variable and can include flaccid or tense bullae, targetoid lesions or lichenoid eruptions. PNP is associated with the following neoplasms (in descending order of frequency): non-Hodgkins lymphoma, chronic lymphocytic leukemia, Castleman �s disease and malignant and benign thymus tumors. Histopathologic findings include s uprabasal acantholysis, dyskeratosis and vacuolar interface with a lichenoid infiltrate. Direct immunofluorescence of  perilesional skin demonstrates intercellular IgG and granular C3 at the dermoepidermal junction. The The  preferred substrate for indirect immunofluorescence is rat bladder and intercellular IgG is found. Target antigens include desmoplakin (250kD), envoplakin (210kD), bullous pemphigoid antigen-1 (230kD),  periplakin (190kD), and desmogleins desmogleins 1 and 3. Many patients succumb to the underlying underlying cancer. Treatment requires management of the malignancy and immunosuppressive agents. Q/Q(M)-477547 Report a Problem Which of the following is NOT associated with Hepatitis C disease? 1

Mixed cryoglobulinemia

2

Uroporphyrinogen decarboxylase deficiency

29

3

Single-stranded RNA viridae

4

Leukocytoclastic vasculitis

5

Single-stranded DNA viridae Q/Q(M)-477560 Report a Problem

Which of the following is NOT associated with Hepatitis C disease? 5

Single-stranded DNA viridae

Hepatitis C virus (HCV) is a single-stranded RNA virus that is a member of the flaviviridae family. Approximately 20-30% of patients develop symptoms with acute infection and 70% will progress to chronic disease. Porphyria cutanea tarda or PCT (uroporphyrinogen decarboxylase deficiency) is associated with HCV in a substantial percentage of patients; in one study, antibodies to HCV were found in 82% of PCT patients. Up to 80% of mixed cryoglobulinemia (MC) cases are associated with HCV. The incidence of lichen planus in HCV patients varies from region to region (0.1-35%). The histopathology of MC lesions is leukocytoclastic vasculitis. Another relatively common association is  polyarteritis nodosa (PAN) which is also related to hepatitis B infection. Finally, generalized pruritis is a common complaint of HCV patients. Q/Q(M)-477560 Report a Problem

A 20-year-old male develops an eruption of 100's of red-brown yellowish papules with involvement of the mucous membranes. He has no lymphadenopathy. The most likely diagnosis is: 1

Montgomery's syndrome

2

Benign cephalic histiocytosis

3

Rosai-Dorfman disease

4

Necrobiotic xanthogranuloma

5

Multicentric reticulohistiocytosis Q/Q(M)-482262 Report a Problem

A 20-year-old male develops an eruption of 100's of red-brown yellowish papules with involvement of the mucous membranes. He has no lymphadenopathy. The most likely diagnosis is: 1

Montgomery's syndrome

This patient has Montgomery's syndrome, or xanthoma disseminatum, which is characterized as a  benign normolipidemic but disfiguring disfiguring condition that demonstrates an eruption of 100's 100's of red-brown yellowish papules and plaques that may involve the mucous membranes. These patients should be evaluated for diabetes insipidus. Benign cephalic histiocytosis shows brownish- yellow papules on the upper face. Rosai-Dorfman, or sinus histiocytosis with massive lymphadenopathy, features  polymorphic papules and plaques plaques in the first two decades with fever, increased ESR, and cervical lymphadenopathy. Necrobiotic xanthogranuloma is characterized by red-orange plaques that may ulcerate. Finally, multicentric reticulohistiocytosis features coral beading around the fingers and is associated with arthritis mutilans and malignancy. Q/Q(M)-482262 Report a Problem

30

The most common location of the lesions in nephrogenic fibrosing dermopathy is: 1

Face

2

Palms and soles

3

Lower extremities

4

Back

5

Chest Q/Q(M)-474546 Report a Problem

The most common location of the lesions in nephrogenic fibrosing dermopathy is: 3

Lower extremities

The most common location for lesions of nephrogenic fibrosis dermopathy is the lower extremities. Q/Q(M)-474546 Report a Problem Which of the following is true regarding piebaldism? 1

It is caused by a mutation in the GJB2 gene

2

It is caused by defective metabolism of phytanic acid

3

It is caused by a defect in a protein subunit of a kinase that activates NFkappaB

4

It is caused by a deficiency of fatty aldehyde dehydrogenase

5

It is caused by a mutation in the proto-oncogene c-KIT Q/Q(M)-473964 Report a Problem

Which of the following is true regarding piebaldism? 5

It is caused by a mutation in the proto-oncogene c-KIT

Piebaldism is caused by an autosomal dominant mutation of the proto-oncogene c-KIT which encodes tyrosine a tyrosine kinase receptor on melanocytes, preventing activation by steel factor. Q/Q(M)-473964 Report a Problem With regards to patients with diabetes mellitus melli tus and skin, which of the following is most accurate? 1

Diabetic dermopathy is most commonly expressed on the forearms and feet

2

Scleredema most often presents on the tibial surfaces

3

Scleredema is the most common skin manifestation of diabetes mellitus

4

Less than 5% of patients with diabetes mellitus have necrobiosis lipoidica

5

Less than 5% of patients with necrobiosis lipoidica have diabetes mellitus Q/Q(M)-480425 Report a Problem

With regards to patients with diabetes mellitus melli tus and skin, which of the following is most accurate? 4

Less than 5% of patients with diabetes mellitus have necrobiosis lipoidica

Patients with diabetes mellitus (DM) may develop thickened skin and tightened joints. Scleredema is 31

occasionally seen in type II diabetics and presents with a peau d�orange texture and appearance on the upper back. Perhaps as many as 20% of those with necrobiosis lipoidica diabeticorum(NLD) have diabetes, but only 0.3% to 3% of diabetics have NLD. The most common skin manifestation of DM is diabetic dermopathy, or skin hyperpigmentation, normally on tibial surfaces. Eruptive xanthomas, which are associated with high triglycerides are occasionally seen as are generalized or perforating granuloma annulare. Q/Q(M)-480425 Report a Problem

Which type of porphyria is the autosomal recessive form of porphyria cutanea tarda? 1

Erythropoietic protoporphyria (EPP)

2

Congenital erythropoietic porphyria (CEP)

3

Hepatoerythropoietic porphyria (HEP)

4

Variegate Porphyria (VP)

5

Acute Intermitent Porphyria (AIP) Q/Q(M)-482194 Report a Problem

Which type of porphyria is the autosomal recessive form of porphyria cutanea tarda? 3

Hepatoerythropoietic porphyria (HEP)

HEP results from deficient, deficie nt, but not absent, activity activit y of uroporphyrinogen decarboxylase (UROD). HEP is the recessive form of familial f amilial PCT. It manifests during infancy or early childhood as  photosensitivity, skin fragility in sun-exposed areas, pink urine, erythrodontia, erythrodontia, and hypertrichosis. Q/Q(M)-482194 Report a Problem Each of the following demonstrates a vasculitis except: 1

Granuloma faciale

2

Henoch-Schoenlein purpura

3

Type 1 cryoglobulinemia

4

Wegener �s granulomatosis

5

Leukocytoclastic vasculitis Q/Q(M)-477357 Report a Problem

Each of the following demonstrates a vasculitis except: 3

Type 1 cryoglobulinemia

In type I cryoglobulinemia, monoclonal IgG or IgM cryoglobulins are found often in association with lymphoma, leukemia, Waldenstrom �s macroglobulinemia, or multiple myeloma. On histopathology, type I cryoglobulinemia is characterized by the deposition of precipitated a morphous cryoglobulins on the endothelium and throughout the vessel wall. The precipitates stain with PAS stain. An inflammatory infiltrate is typically lacking in contrast to mixed cryoglobulinemia (which shows a leukocytoclastic vascultitis) Q/Q(M)-477357 Report a Problem 32

What is the treatment of choice for porphyria cutanea tarda? 1

Phlebotomy

2

Antimalarials

3

Erythropoietin

4

Oral iron supplementation

5

Naproxen Q/Q(M)-482567 Report a Problem

What is the treatment of choice for porphyria cutanea tarda? 1

Phlebotomy

Porphyria cutanea tarda (PCT) is caused by a deficienc y in the enzyme uroporphyrinogen decarboxylase. Phlebotomy is the treatment of choice and may improve enz ymatic activity by removing iron, an inhibitor of the enzyme. Antimalarials and erythropoietin are alter native therapies. Oral iron supplementation may worsen PCT. Naproxen is a common cause of pseudoporphyria. Q/Q(M)-482567 Report a Problem

Squamous cell carcinoma is seen in which syndrome? 1

Gorlin syndrome

2

Rombo syndrome

3

Nicolau-Balus syndrome

4

Rasmussen syndrome

5

Bazex syndrome (Acrokeratosis paraneoplastica) Q/Q(M)-477343 Report a Problem

Squamous cell carcinoma is seen in which syndrome? 5

Bazex syndrome (Acrokeratosis paraneoplastica)

Basex syndrome, also Acrokeratosis paraneoplastica, presents with symmetric erythematous, nearly violaceous, psoriasiform dermatoses of the hands, feet, e ars and nose. The syndrome is nearly associated with an underlying malignancy, usually squamous cell carcinoma of the upper aerodigestive tract. Q/Q(M)-477343 Report a Problem The presence of antibodies to c-ANCA is characteristically seen in patients with which disease? 1

Ulverative colitis

2

Churg-Strauss disease 33

3

Polyarteritis nodosa

4

Wegener �s granulornatosis

5

Nodular vasculitis Q/Q(M)-477362 Report a Problem

The presence of antibodies to c-ANCA is characteristically seen in patients with which disease? 4

Wegener �s granulornatosis

Wegener �s granulomatosis is a necrotizing granulomatous disorder that most commonly affects t he upper and lower respiratory tracts, kidneys, and eye. Cytoplasmic pattern antineutrophil cytoplasmic autoantibody (anti -proteinase-3) is often positive in this disease. Churg-Strauss is associated with a  positive p-ANCA. Q/Q(M)-477362 Report a Problem Erythema gyratum repens is known to be associated with all of the following malignancies except: 1

Lung carcinoma

2

Breast carcinoma

3

Cervical carcinoma

4

Bladder carcinoma

5

Gastric carcinoma Q/Q(M)-473956 Report a Problem

Erythema gyratum repens is known to be associated with all of the following malignancies except: 5

Gastric carcinoma

Erythema gyratum repens presents more commonly in men than in women, and appears clinically as concentric erythematous rings with trailing scale on the trunk and proximal extremities. The skin is described as having a "wood grain" appearance. Skin findings often precede the diagnosis of an associated malignancy. Lung carcinoma is the most commonly associated malignancy, but it has also  been reported in association with breast, cervical, bowel, and bladder cancer. Q/Q(M)-473956 Report a Problem Which of the following statements regarding porphyrias is TRUE? 1

Elevated uroporphyrins are found in the red blood cells of hepatoerythropoietic porphyria

2

Delta aminolevulenic acid is the only oxidized porphyrin

3

Acute intermittent porphyria is the most common form of porphyria

4

Griseofulvin is safe for those with variegate porphyria

5

Plasma fluoresces at 410 nm in patients with variegate porphyria Q/Q(M)-477556 Report a Problem

Which of the following statements regarding porphyrias is TRUE? 34

2

Delta aminolevulenic acid is the only oxidized porphyrin

Acute intermittent porphyria (AIP) is the second most common porphyria and is caused by a deficiency in porphobilinogen (PBG) deaminase, which is located in the cytosol. Patients suffer from colick y pain,  paralysis and psychiatric disorders. There are no specific skin manifestations. PBG and aminolevulenic acid (ALA) are elevated in the urine. Attacks are precipitated by medications such as barbiturates, estrogen, griseofulvin, and sulfonamides as well as starvation, fever and infection. Treatment includes glucose loading and hematin infusion. Congenital er ythropoietic porphyria (CEP) or G �nter �s disease is caused by a defect in uroporphyrinogen III synthase, which is found in the cytosol. Patients are extremely photosensitive and erythema, blistering and scarrin g result. Patients present with red urine early in life along with hypertrichosis and red -stained teeth that fluoresce. Uroporphyrins (URO) are much high than coproporphyrins (COPRO) in the urine. URO is found in the red blood cells (rbcs) and COPRO is found in the stool. The rbcs display stable fluorescence. Porphyria cutanea tarda (P CT) is the most common porphyria and is caused by a deficiency (usually sporadic) in uroporphyrinogen decarboxylase, which is found in the cytosol. Patients present with photosensitivity and blistering of sun-exposed areas, especially the dorsal hands. Hypertrichosis and scle rodermoid changes may occur as well. Liver disease (hepatitis C or alcoholic cirrhosis) is often present and hemochromatosis may be associated. Urine may fluoresce pink or coral-red coral -red with Wood�s lamp. URO>COPRO in the urine and low levels of COPRO are found in the stool. Treatments include phlebotomy, antimalarials, and therapy for liver disease if appropriate. Hereditary coproporphyria (HCP) is caused by a deficiency in coproporphyrinogen oxidase, which is found in the mitochondria. One-third of patients are  photosensitive, and patients suffer gastrointestinal and neurological symptoms similar to AIP. Urine Urine COPRO is elevated only with attacks, and COPRO is present in the stool. Variegate porphyria is the result of decreased activity of protoporphyrinogen oxidase, which is present in the mitochondria. It combines the skin lesions of PCT with the systemic manifestations of AIP. Urine COPRO:URO is 1:1 or COPRO> URO to distinguish it from PCT, and PROTO is found in the stool. The plasma fluoresces at 626nm. Precipitators and treatments are similar to AIP. Erythropoietic protoporphyria (EPP) is caused by ferrochetalase deficiency, which is present in the mitochondria. Patients experience immediate burning of the skin s kin with sun exposure. Protoporphyrin IX, the only oxidized porphyrin in the heme pathway and absorbs in the Soret band (400-410nm). Patients have erythematous plaques in a  photo-distribution. Urine porphyrins porphyrins are normal. PROTO is found in the rbcs and the stool. Excessive  porphyrins deposited in the liver lead to gallstones gallstones and cirrhosis. Beta carotene may helpful. Hepatoerythropoietic porphyria (HEP) is essentially a homozygous form of PCT, with deficiency in uroporphyrinogen decarboxylase. decarboxylase. It is clinically clinical ly similar to CEP with red urine and hypertrichosis, vesicles and scarring of sun-exposed skin. URO is present i n the urine and COPRO in the stool. PROTO is present in rbcs which distinguishes it from CEP, which was URO in rbcs. Q/Q(M)-477556 Report a Problem

Which of the following may be associated with Graves � disease? 1

Dermatitis herpetiformis

2

Geographic tongue

3

Hypohidrosis

4

Madarosis

5

Thick, pale lips Q/Q(M)-477561 Report a Problem

Which of the following may be associated with Graves � disease? 1

Dermatitis herpetiformis 35

Graves� disease is a thyrotoxic condition that results from the production of thyroid-stimulating immunoglobulins (TSI) by stimulated B lymphocytes. The TSI bind t o the thyroid-stimulating hormone (TSH) receptor and mimic TSH thereby stimulating thyroid growth and thyroid hormone overproduction. Signs and symptoms of Graves � disease include goiter, tachycardia, exophthalmos, tremor, sweating, palpitations, smooth moist skin, diarrhea, sleeplessness, ir ritability, and weight loss. Autoimmune cutaneous disease may also be associated wit h Graves� disease including vitiligo, dermatitis herpetiformis, herpes gestationis, and pemphigus vulgaris. Cutaneous manifestations of hypothyroidism include xerosis, hyperhidrosis, yellowish hue, myxedema, and purpura. The hair may  be dry, brittle and coarse; alopecia may be diffuse and/or involve the the lateral eyebrow (madarosis). Q/Q(M)-477561 Report a Problem Which of the following is NOT true regarding Cushing's Syndrome? 1

Urine cortisol levels are elevated

2

Corticotropin is suppressed with the administration of dexamethasone

3

It may be associated with an underlying oat cell lung carcinoma

4

It may be associated with hypertension and hypokalemia

5

May present with hyperpigmentation and facial plethora Q/Q(M)-473957 Report a Problem

Which of the following is NOT true regarding Cushing's Syndrome? 2

Corticotropin is suppressed with the administration of dexamethasone

In Cushing's Syndrome, corticotropin is NOT suppressed with the administration of dexamethasone. The other statements are true. Q/Q(M)-473957 Report a Problem A patient with gluten-sensitive enteropathy presents with vesicles on the extensor surfaces of the extremities. What findings are most likely on a perilesional biopsy? 1

Granular IgA at the dermoepidermal junction on direct immunofluorescence

2

Linear C3 and IgG at the dermoepidermal junction on direct immunofluorescence

3

Linear IgA surrounding vessels on direct immunofluorescence

4

Pautrier �s micro-abscesses in the epidermis on H & E

5

Leukocytoclastic vasculitis Q/Q(M)-477552 Report a Problem

A patient with gluten-sensitive enteropathy presents with vesicles on the extensor surfaces of the extremities. What findings are most likely on a perilesional biopsy? 1

Granular IgA at the dermoepidermal junction on direct immunofluorescence

This patient has dermatitis herpetiformis (DH). DH is characterized by itchy papulovesicles on extensor surfaces of the extremities. Neutrophilic infiltrates at the dermal papillae with vesicle formation are found on histopathologic examination. On immunofluorescence, granular IgA (direct ed against 36

transglutaminase) deposits are found in perilesional skin. Over 90% of patients have gluten-sensitive enteropathy of varying severity. Dapsone is almost universally therapeutic for the skin disease, but not the enteropathy. A gluten free diet, although difficult to maintain, treats the enteropathy. The majority of DH patients have the HLA class II DQ2 genotype. On indirect immunofluorescence, one may find antigliadin, antiendomyseal or antireticulin antibodies. Autoimmune thyroid disease (especially Hashimoto�s thyroiditis), enteropathy-associated T-cell non-Hodgkin �s lymphoma and insulindependent diabetes are the most common autoimmune associations. Pautrier �s micro-abscesses are found in mycosis fungoides. Leukocytoclastic vasculitis is not a feature of DH. Q/Q(M)-477552 Report a Problem

A complication seen in women of childbearing age with the condition in the figure is: 1

Atrial septal defect

2

Pulmonary lymphangioleiomyoma

3

Gastrointestinal bleeding

4

Lisch nodules

5

Pulmonary emboli Q/Q(M)-474944 Report a Problem

A complication seen in women of childbearing age with the condition in the figure is: 2

Pulmonary lymphangioleiomyoma

This picture demonstrates a shagreen patch in a pa tient with Tuberous sclerosis. A complication occurring in female patients of child-bearing age i s pulmonary lymphangioleiomyoma. Q/Q(M)-474944 Report a Problem

Which of the following syndromes necessitates a work-up for colon cancer ? 1

Turcot

2

Birt-Hogg-Dube

3

LAMB

4

Muckle Wells

5

Wells Q/Q(M)-482459 Report a Problem

37

Which of the following syndromes necessitates a work-up for colon cancer ? 1

Turcot

Turcot syndrome is a variant of familial adenomatous polyposis, FAP with the same mutation - APC gene, characterized by FAP and brain cancer. Patients may ma y also exhibit other clinical features of Gardner's syndrome. Both patients with Gardner's Syndrome and Turcot Syndrome necessitate a colon cancer work-up. Q/Q(M)-482459 Report a Problem

Which of the following is true regarding cutaneous associations with hepatitis C virus (HCV) infection? Erosive mucosal lichen planus has a weaker association with HCV than does cutaneous lichen 1  planus 2

Pruritus in the setting of chronic HCV infection is generally correlated with elevated bile salt levels in the setting of liver failure

3

Polyarteritis nodosa is associated with HCV infection but not hepatitis B virus infection

4

HCV-related porphyria cutanea tarda may be caused by decompartmentaliz ation of iron stores and resultant oxidation of uroporphyrinogen decarboxylase

5

Cutaneous reactions to HCV treatment are less common with interferon/ribavirin combination treatment than with treatment with interferon alone Q/Q(M)-473930 Report a Problem

Which of the following is true regarding cutaneous associations with hepatitis C virus (HCV) infection? 4

HCV-related porphyria cutanea tarda may be caused by decompartmentalizati on of iron stores and resultant oxidation of uroporphyrinogen uroporphyrinogen decarboxylase

Erosive mucosal variant of lichen planus has the strongest association with HCV. The pathogenesis of  pruritus in the setting of chronic HCV infection may be related to elevated bile salt levels in i n the setting of liver failure, but there is not always a direct correlation between serum bil e salt level and degree of  pruritus. Polyarteritis nodosa is associated with both HCV infection and hepatitis B virus infection. Statement D is correct. Cutaneous reactions reacti ons to HCV treatment are more common with interferon/ribavirin combination treatment than with treatment with interferon alone. Q/Q(M)-473930 Report a Problem

A 55 year old patient presents with new onset brown macules on arms, legs, face and palms. She gives a 3 month history of diarrhea, abdominal cramps, weight loss and protein-losing enteropathy. The most likely diagnosis is: 1

Peutz-Jeghers syndrome

2

Cowden disease

3

Ulcerative colitis

4

Cronkhite-Canada syndrome

5

Plummer-Vinson syndrome Q/Q(M)-479610 Report a Problem

A 55 year old patient presents with new onset brown macules on arms, legs, face and palms. She gives

38

a 3 month history of diarrhea, abdominal cramps, weight loss and protein-losing enteropathy. The most likely diagnosis is: 4

Cronkhite-Canada syndrome

Cronkhite-Canada is an aquired disease characteri zed by the development of polyps throughout the GI tract. Patients can present with hyperpigmented macules as well as the sequella of GI malabsorption. The malignant transformation of polyps can occur. Q/Q(M)-479610 Report a Problem

Regarding carcinoid syndrome, which of the following is TRUE? 1

Symptoms are caused by metastases to the skin

2

The most common location for the tumor is the rectum

3

Somatostatin is a preferred treatment

4

VMA is elevated in the urine

5

The tumors should not be removed Q/Q(M)-477559 Report a Problem

Regarding carcinoid syndrome, which of the following is TRUE? 3

Somatostatin is a preferred treatment

Carcinoid syndrome is caused by a tumor originating in the endocrine argentaffin cells. In order of descending frequency, the most common locations are appendix > small bowel > rectum. Patients experience facial flushing that descends anatomically, diarrhea, hyperhidrosis and bronchial reactivity. A pellagra-like eruption may occur as well as telangiectasia and sclerodermoid changes. Patients experience symptoms once the disease metastasizes to the liver or if it did not involve the GI tract initially. Diagnosis can be made finding elevated 5-hydroxyindolacetic acid (5-HIAA) in the urine.  Nitrosonaphthol turns urine purple if 5-HIAA levels are significantly elevated, and can be used as a screening test. Bananas, tomatoes, plums, avocadoes and eggplant can incre ase levels of 5-HIAA. Treatment involves removal of the tumor along with medical therapy if appropriate. Agents us ed include somatostatin, methylsergide, cyproheptadine, and beta blockers. Vanillylmandelic acid (VMA) is an important urinary metabolic product of epinephrine and norepinephrine and is detected in the diagnosis of pheochromocytoma. Q/Q(M)-477559 Report a Problem

An uncommon complication of treatment with potassium iodide is: 1

Acute generalized exanthematous pustulosis

2

Erythema nodosum

3

Wolff-Chiakoff effect

4

Exacerbation of lichen planus

5

Hyperhidrosis Q/Q(M)-474547 Report a Problem

39

An uncommon complication of treatment with potassium iodide is: 3

Wolff-Chiakoff effect

An uncommon complication of treatment with potassium iodide is the Wolff-Chiakoff effect. Q/Q(M)-474547 Report a Problem

Which of the following is true about blue rubber bleb nevus syndrome? 1

typically appears in adulthood

2

the venous malformations regress with time

3

characterized by compressible blue nodules 0.1 to 5 cm in size.

4

the nodules are pruritic

5

the gastrointestinal lesions are typically located in the stomach Q/Q(M)-482482 Report a Problem

Which of the following is true about blue rubber bleb nevus syndrome? 3

characterized by compressible blue nodules 0.1 to 5 cm in size.

Blue rubber bleb nevus is associated with soft and compressible blue nodules 0.1 to 5 cm in size. The cutaneous lesions typically appear in childhood, and they do not regress with time. The venous malformations are mainly asymptomatic but can be painful when thrombosis occurs. The venous malformations also involve the gastrointestinal tract, most commonly the small bowel, where they are friable and can bleed. Q/Q(M)-482482 Report a Problem

Which of the following statements about multiple endocrine neoplasia (MEN) syndromes is FALSE? 1

MEN I is associated with angiofibromas and collagenomas

2

MEN IIa is related to a defect in RET proto-oncogene

3

MEN IIa and IIb are both associated with medullary thyroid carcinoma

4

MEN I, IIa and IIb are all inherited in an autosomal dominant fashion

5

MEN IIa is associated with multiple mucosal neuromas Q/Q(M)-477549 Report a Problem

Which of the following statements about multiple endocrine neoplasia (MEN) syndromes is FALSE? 5

MEN IIa is associated with multiple mucosal neuromas

Multiple endocrine neoplasia (MEN) syndromes are divided into types I, IIa and IIb. All are autosomal dominantly inherited. Type I is associated with mutations in MENI; t ype IIa and IIb involve mutations of RET, which encodes a tyrosine kinase receptor. Type I has the following cutaneous features: facial angiofibromas, collagenomas, lipomas, hypopigmented macules and caf �-au-lait macules. Type IIa displays macular amyloidosis, while IIb features multi ple mucosal neuromas. Systemic features of type I include peptic ulcer disease (as ( as part of Zollinger-Ellison syndrome), s yndrome), parathyroid hyperplasia hyperplasia or adenoma, pancreatic tumors and pituitary neoplasia. Type IIa is charact erized by Zollinger-Ellison 40

syndrome, medullary thyroid carcinoma, pheochromocytoma, and parathyroid neoplasia. Type IIb displays marfanoid habitus, Hirschsprung disease, medulloblastoma, medullary thyroid carcinoma,  pheochromocytoma and ocular neuromas. neuromas. Q/Q(M)-477549 Report a Problem

This syndrome is cause by defects in the genes tha t code for tumor suppressor proteins hamartin and tuberin. 1

Bannayan-Riley-Rubalcaba Syndrome

2

Blue Rubber Bleb Nevus Syndrome

3

Cronkhite-Canada Syndrome

4

Bourneville's Disease

5

Cowden's Syndrome

Q/Q(M)-473935 Report a Problem This syndrome is cause by defects in the genes tha t code for tumor suppressor proteins hamartin and tuberin. 4

Bourneville's Disease

Tuberous sclerosis, also known as Bourneville's Disease, is a n autosomal dominant neurocutaneous disorder due to mutations in the TSC1 and TSC2 genes which code for the tumor suppressor proteins hamartin and tuberin, respectively. Q/Q(M)-473935 Report a Problem

Hepatitis C infection is associated with: 1

Gianotti-Crosti syndrome

2

Mixed cryoglobulinemia

3

Pityriasis rosea

4

Kapsi�s sarcoma

5

Oral hairy leukoplakia Q/Q(M)-477124 Report a Problem

Hepatitis C infection is associated with: 2

Mixed cryoglobulinemia

Cutaneous manifestation associated with hepatitis C include necrolytic acral erythema, porphyria cutanea tarda, lichen planus, polyarteritis nodosa and mixed cryoglobulinemia. Mixed cryoglobulinemias is a systemic vasculitis with variable manifestations including palpable purpura, arthralgias and weakness. Q/Q(M)-477124 Report a Problem

The clinical findings in figure 3 are associated with which disorder? 41

1

Graves disease

2

Myasthenia gravis

3

Dermatomyositis

4

Sarcoidosis

5

Rheumatoid Arthritis Q/Q(M)-475405 Report a Problem

The clinical findings in figure 3 are associated with which disorder? 1

Graves disease

This slide shows pretibial myxedema in a patient with Grave �s disease. Q/Q(M)-475405 Report a Problem

Which of the following statements regarding porphyrias is FALSE? 1

Fecal coproporphyrinogen is increased in variegate porphyria

2

Hepatoerythropoietic porphyria is the homozygous form of porphyria cutanea tarda

3

Acute intermittent porphyria has no skin findings

4

Coproporphyrinogen Coproporphyrinogen is elevated more than uroporphyrinogen in 24 hour urine samples in  porphyria cutanea tarda

5

In erythropoietic protoporphyria, protoporphyrin IX absorbs in the Soret band Q/Q(M)-477555 Report a Problem

Which of the following statements regarding porphyrias is FALSE? 4

Coproporphyrinogen Coproporphyrinogen is elevated more than uroporphyrinogen in 24 hour urine samples in 42

 porphyria cutanea tarda Acute intermittent porphyria (AIP) is the second most common porphyria and is caused by a deficiency in porphobilinogen (PBG) deaminase, which is located in the cytosol. Patients suffer f rom colicky pain,  paralysis and psychiatric disorders. There are no specific skin manifestations. PBG and aminolevulenic acid (ALA) are elevated in the urine. Attacks are precipitated by medications such as barbiturates, estrogen, griseofulvin, and sulfonamides as well as starvation, fever and infection. Treatment includes glucose loading and hematin infusion. Congenital er ythropoietic porphyria (CEP) or G �nter �s disease is caused by a defect in uroporphyrinogen III synthase, which is found in the cytosol. Patients are extremely photosensitive and erythema, blistering and scarring result . Patients present with red urine early in life along with hypertrichosis and red -stained teeth that fluoresce. Uroporphyrins (URO) are much high than coproporphyrins (COPRO) in the urine. URO is found in the red blood cells (rbcs) and COPRO is found in the stool. The rbcs display stable fluorescence. Porphyria cutanea tarda (P CT) is the most common porphyria and is caused by a deficiency (usually sporadic) in uroporphyrinogen decarboxylase, which is found in the cytosol. Patients present with photosensitivity and blistering of sun-exposed areas, especially the dorsal hands. Hypertrichosis and scle rodermoid changes may occur as well. Liver disease (hepatitis (hepatiti s C or alcoholic cirrhosis) is often present and a nd hemochromatosis may be associated. Urine may fluoresce pink or coral-red coral -red with Wood�s lamp. URO>COPRO in the urine and low levels of COPRO are found in the stool. Treatments include phlebotomy, antimalarials, and therapy for liver disease if appropriate. Hereditary coproporphyria (HCP) is caused by a deficiency in coproporphyrinogen oxidase, which is found in the mitochondria. One-third of patients are  photosensitive, and patients suffer gastrointestinal and neurological symptoms similar to AIP. Urine COPRO is elevated only with attacks, and COPRO is present in the st ool. Variegate porphyria is the result of decreased activity of protoporphyrinogen oxidase, which is present in the mitochondria. It combines the skin lesions of PCT with the systemic manifestations of AIP. Urine COPRO:URO is 1:1 or COPRO> URO to distinguish it from PCT, and PROTO is found in the stool. The plasma fluoresces at 626nm. Precipitators and treatments are similar simi lar to AIP. Erythropoietic protoporphyria (EPP) is caused by ferrochetalase deficiency, which is pres ent in the mitochondria. Patients experience immediate burning of the skin s kin with sun exposure. Protoporphyrin IX, the only oxidized porphyrin in the heme pathway and absorbs in the Soret band (400-410nm). Patients have erythematous plaques in a  photo-distribution. Urine porphyrins porphyrins are normal. PROTO is found in the rbcs and the stool. Excessive  porphyrins deposited in the liver lead to gallstones gallstones and cirrhosis. Beta carotene may helpful. Hepatoerythropoietic porphyria (HEP) is essentially a homozygous form of PCT, with deficiency in uroporphyrinogen decarboxylase. decarboxylase. It is clinically clinical ly similar to CEP with red urine and hypertrichosis, vesicles and scarring of sun-exposed skin. URO is present in the urine and COPRO in the stool. PROTO is present in rbcs which distinguishes it from CEP, which was URO in rbcs. Q/Q(M)-477555 Report a Problem

What is the unique laboratory finding in stool in porphyria cutanea tarda (PCT) and hepatoerythropoietic porphyria (HEP)? 1

elevated coproporphyrins

2

elevated isocoproporphyrins

3

decreased isocoproporphyrins

4

elevated protoporphyrins

5

elevated uroporphyrins Q/Q(M)-482486 Report a Problem

What is the unique laboratory finding in stool in porphyria cutanea tarda (PCT) and hepatoerythropoietic porphyria (HEP)? 43

2

elevated isocoproporphyrins

PCT and HEP are associated with elevated isocoproporphyrins in the stool. Congenital erythropoietic  porphyria, hereditary coproporphyria coproporphyria is associated with increased coproporphyria in the stool, and elevated stool proporphyria is seen in variegate porphyria. Q/Q(M)-482486 Report a Problem

All of the following statements are true regarding this condition EXCEPT: 1

Diabetes or glucose intolerance is found in 20% of these patients

2

This condition may be associated with cutaneous anesthesia, hypohidrosis, and partial alopecia

3

There is no impact of tight glucose control on the likelihood of developing this condition

4

0.3-3% of diabetics have this skin condition

5

This condition is associated with increased dermal mucin Q/Q(M)-474080 Report a Problem

All of the following statements are true regarding this condition EXCEPT: 5

This condition is associated with increased dermal mucin

 Necrobiosis Lipoidica Diabeticorum (NLD) is found in 0.3-3% 0.3-3% of diabetics. Approximately 20% of of  NLD patients have diabetes or glucose intolerance. It presents with single or multiple red-brown  papules which progress to sharply demarcated yellow-brown atrophic, atrophic, telangiectatic plaques with violaceous, irregular borders; common sites include the shins. Cutaneous anesthesia, hypohidrosis, and  partial alopecia can be found. Pathology shows shows palisading granulomas containing degenerating collagen (necrobiosis); with NO increase in derma l mucin. There is no impact of tight glucose control on the likelihood of developing NLD. Q/Q(M)-474080 Report a Problem

44

The organism that causes this infection shown in the figure is: 1

T. tonsuran

2

C. immitis

3

C. albicans

4

P. aeruginoas

5

T. verrucosum Q/Q(M)-474936 Report a Problem

The organism that causes this infection shown in the figure is: 3

C. albicans

The organism that causes erosio interdigitalis blastomycetica is candida albicans. Q/Q(M)-474936 Report a Problem

A patient infected with the hepatitis C virus is at highest risk for which of the following cutaneous manifestations? 1

Increased hyaluronic acid in dermis of lower and upper extremities

2

Lindsay�s nails

3

Congenital hyperpigmentation of the retinal pigment (CHRPE)

4

Periorificial lentigines

5

Positive rheumatoid factor and decreased complement (C3) Q/Q(M)-480423 Report a Problem

A patient infected with the hepatitis C virus is at highest risk for which of the following cutaneous manifestations? 5

Positive rheumatoid factor and decreased complement (C3)

Patients with hepatitis C virus (HCV) may experience numerous cutaneous manifesta tions of their internal disease. Cryoglobulinemia (CG) may result in small vessel leukocytoclastic vasculitis. Types of CG include: type I, which is monoclonal Ig; type II, which is pol yclonal IgG and monoclonal IgM; type III, which is polyclonal IgG and polyclonal IgM. Mixed CG is the most commonly associated type of CG with HCV. Patients often have a positive rhematoid factor and have C3 complement decreased. Other diseases associated with wit h HCV include porphyria cutanea tarda, lichen planus and polyarteritis nodosa. Thyroid-related dermopathy results in increased hyaluronic acid in the dermis. Lindsay's nails , or half and half nails, are associated with renal disease. Congenital hyperpigmentation of the retinal 45

 pigment (CHRPE) is seen in Gardner's syndrome. Peutz-Jeghers, Peutz-Jeghers, among other conditions presents with  periorificial lentigines. Q/Q(M)-480423 Report a Problem The most common autoimmune disease associated with hepatitis C is: 1

Autoimmune thyroiditis

2

Myasthenia gravis

3

Aplastic anemia

4

Lymphocytic sialadenitis

5

Rheumatoid arthritis Q/Q(M)-474548 Report a Problem

The most common autoimmune disease associated with hepatitis C is: 1

Autoimmune thyroiditis

The most common autoimmune disease seen in association with Hepatitis C is autoimmune thyroiditis. Q/Q(M)-474548 Report a Problem

Which of the following is (are) characteristic of basal cell nevus syndrome? 1

Autosomal recessive inheritance

2

NEMO gene mutation

3

Tram-track calcifications

4

Colobomas

5

Telangiectasias Q/Q(M)-479280 Report a Problem

Which of the following is (are) characteristic of basal cell nevus syndrome? 4

Colobomas

Basal cell nevus syndrome (Gorlin syndrome) is caused by an autosomal dominant mutation in PTCH gene that encodes PTC protein involved in sonic hedge hog pathway. This mutation leads to loss of inhibition of smoothened (SMO) leading to an increased expression of other genes. Cutaneous findings include nevoid basal cell carcinomas, milial cysts, epidermoid cysts and palmar-plantar pits.  Neurological findings include calcifications of the falx cerebri, agenesis of the corpus callosum, mental retardation, medulloblastoma and spina bifida. Ocular findings include blindness, cataracts, colobomas and strabismus. Other findings include odontogenic cysts of the jaw, frontal bossing, bifid ribs, and  pectus deformity. NEMO gene mutations are found in incontinentia pigmenti. Tram-track calfications are seen Sturge-Weber syndrome. Telangiectasias are seen in ataxia-telangiectasia among others. Q/Q(M)-479280 Report a Problem

When metastases to the skin occur from a thyroid malignancy, they are usually due to: 1

Medullary carcinoma 46

2

Papillary adenocarcinoma

3

Follicular carcinoma

4

Anaplastic carcinoma

5

Cutaneous metastases have not been reported in association with thyroid malignancies Q/Q(M)-473931 Report a Problem

When metastases to the skin occur from a thyroid malignancy, they are usually due to: 2

Papillary adenocarcinoma

Metastases to the skin from a thyroid malignancy are rare, but most reported cases occur with papillary adenocarcinoma. Q/Q(M)-473931 Report a Problem Hypothyroidism may result in all of the following cutaneous findings except: 1

Increase in the percentage of telogen hairs

2

Yellowish hue

3

Purpura

4

Madarosis

5

All of these answers are correct Q/Q(M)-473945 Report a Problem

Hypothyroidism may result in all of the following cutaneous findings except: 5

All of these answers are correct

All of the above are non-specific cutaneous manifestations of hypothyroidism: an increase in  percentage of telogen hairs, yellowish hue caused by carotenemia, purpura purpura secondary to impaired wound healing, and loss of the lateral third of the eyeborw (madarosis). Q/Q(M)-473945 Report a Problem

All of the following are true regarding Cockayne Syndrome EXCEPT: 1

Inheritance is autosomal recessive

2

It is caused by an inability to repair cyclobutane dimers

3

It is associated with basal ganglia calcifications

4

It is associated with retinal pigment degeneration with a "salt and pepper" appearance

5

None of these answers are incorrect Q/Q(M)-473970 Report a Problem

All of the following are true regarding Cockayne Syndrome EXCEPT: 47

5

None of these answers are incorrect

Cockayne Syndrome is an autosomal recessive disorder caused by a mutation of an unknown gene that results in the inability to repair c yclobutane dimers induced by UV exposure. Cutaneous findings include photosensitivity, "bird-headed" facies, and "Mickey Mouse" ears. Stateme nts A-D are true regarding the syndrome. Q/Q(M)-473970 Report a Problem

A 70-year old female develops erythema with fine adherent scale on acral skin that progresses to keratoderma and eventually a more generalized psoriasiform dermatitis . What is the most likely likel y underlying malignancy? 1

Squamous cell carcinoma of the larynx

2

Adenocarcinoma of the colon

3

Adenocarcinoma of the breast

4

Squamous cell carcinoma of the vagina

5

Medullary thyroid carcinoma

Q/Q(M)-482239 Report a Problem A 70-year old female develops erythema with fine adherent scale on acral skin that progresses to keratoderma and eventually a more generalized psoriasiform dermatitis . What is the most likely likel y underlying malignancy? 1

Squamous cell carcinoma of the larynx

Acrokeratosis Paraneoplastica (Bazek Syndrome) is always associate d with malignancy. Most commonly, the associated malignancy is a squamous cell carcinoma of the upper aerodigestive tract. The skin disease usually follows the course of t he malignancy. Q/Q(M)-482239 Report a Problem

A patient presents with mild mental retardation, infertility, joint contractures, short stature, ichthyosis, and sparse hair with trichoschisis. All of the following are true regarding this patient's condition EXCEPT: 1

This syndrome is inherited in an autosomal recessive manner

2

If photosensitivity is a feature, gonad size may be normal

3

Patients may have associated cataracts

4

The syndrome is caused by impaired nucleotide excision repair

5

Perifoveal glistening white dots are a feature Q/Q(M)-473971 Report a Problem

A patient presents with mild mental retardation, infertility, joint contractures, short stature, ichthyosis, and sparse hair with trichoschisis. All of the following are true regarding this patient's condition EXCEPT: 48

5

Perifoveal glistening white dots are a feature

The patient described has IBIDS syndrome (Ichthyosis, Brittle hair, Intellect ual impairment, Decreased fertility, and Short stature), or Tay's syndrome. This syndrome is autosomal recessive, caused by mutations in the ERCC2/XPD or ERCC3/XPB genes, resulting in impaired nucleotide excision repair. Cataracts may be a feature. If photosensitivity is a feature (PIBIDS), gonal siz e is normal. Perifoveal glistening white dots are a feature of Sjorgen-Larsson Sjor gen-Larsson Syndrome, not IBIDS. Q/Q(M)-473971 Report a Problem

A patient presents with diffuse waxy keratoderma of the palms and soles as well as oral hairy leukoplakia. Which of the following statements regarding this condition is NOT true? 1

This condition may be associated with esophageal carcinoma

2

This condition is autosomal recessive

3

Family members should be advised to undergo cancer screening

4

Features may include squamous cell carcinomas arising from keratodermic skin

5

None of these answers are correct (all are true) Q/Q(M)-473961 Report a Problem

A patient presents with diffuse waxy keratoderma of the palms and soles as well as oral hairy leukoplakia. Which of the following statements regarding this condition is NOT true? 2

This condition is autosomal recessive

Howell-Evans syndrome is an autosomal dominant disorder that presents with diffuse waxy keratoderm of the palms and soles, as well as oral hairy leukoplakia and squamous cell carcinoma arising from keratodermic skin. It is associated with esophageal carcinoma; cancer screening in family members is advised. Q/Q(M)-473961 Report a Problem A patient with end stage renal disease undergoes an MRI examination with contrast. He subsequently develops woody indurated plaques on the extremities. Fatalities seen in nephrogenic systemic fibrosis are due to what underlying process? 1

Fibrosis of respiratory muscles

2

Fibrosis of cardiac smooth muscle

3

Fibrosis of the gastrointestinal tract

4

Fibrosis of cerebral arteries

5

Fibrosis of the liver Q/Q(M)-482737 Report a Problem

A patient with end stage renal disease undergoes an MRI examination with contrast. He subsequently develops woody indurated plaques on the extremities. Fatalities seen in nephrogenic systemic fibrosis are due to what underlying process? 49

1

Fibrosis of respiratory muscles

 Nephrogenic systemic fibrosis is a chronic fibrosing disorder seen most commonly in the setting of renal failure. There is a reported association also with gadolinium as contrast in an MRI. Patients  present with woody indurated plaques plaques or nodules primarily of the extremities, with occasional involvement of the trunk. Fulminant and fatal disease is rare, occurring in about 5% of cases. Death is due to impaired ventilation secondary to fibrosis of respiratory muscles. Q/Q(M)-482737 Report a Problem A patient presents with a complaint of facial flushing that spreads to the neck and upper trunk. Review of systems reveals that the patient has occassional bouts of diarrhea, and intermittent bronchospasm. Laboratory testing reveals an elevated urine 5-hydroxyindole-acetic acid level. Which of the statements regarding this condition is NOT correct? 1

Chest and abdominal/pelvic CT scanning should be the next step in this patient's evaluation

2

The patient may have an associated sclerodermoid-like eruption on examination

3

The patient likely has a neoplasm originating in the endocrine argentaffin cells

4

Treatment with cyproheptadine would be contraindicated

5

The patient likely has a neoplasm located in the GI tract Q/Q(M)-481616 Report a Problem

A patient presents with a complaint of facial flushing that spreads to the neck and upper trunk. Review of systems reveals that the patient has occassional bouts of diarrhea, and intermittent bronchospasm. Laboratory testing reveals an elevated urine 5-hydroxyindole-acetic acid level. Which of the statements regarding this condition is NOT correct? 4

Treatment with cyproheptadine would be contraindicated

This patient has carcinoid sydrome, which presents with facial flushing, diarrhea, and intermittent  bronchospasm. Patients can also develop telangiectasia, pellagra-like, or sclerodermoid-like cutaneous findings. This syndrome is caused by a neoplasm originating in the endocrine argentaffin cells. 80 -85% are found in the GI tract. Treatment is surgical removal of the tumor, or medical treatment with medications including somatostatin, methylsergide, cyproheptadine, beta-blockers, and phenothiazine derivatives. Q/Q(M)-481616 Report a Problem

Which one of the following clinical findings suggests the diagnosis of multiple myeloma? 1

Sudden eruption of seborrheic keratoses

2

Hyperkeratotic follicular papules on the nose

3

Erythema, vesicles, and erosions in periorifical and acral areas

4

Concentric erythematous rings with trailing scale on trunk and extremities

5

Tripe palms Q/Q(M)-482196 Report a Problem

Which one of the following clinical findings suggests the diagnosis of multiple myeloma? 50

2

Hyperkeratotic follicular papules on the nose

Hyperkeratotic follicular nasal papules are associated with multiple myeloma. A sudden increase in number and size of seborrheic keratoses is called cal led the sign of Lesser-Trelat Lesser-Trela t and can be seen in gastric or colon carcinoma. Erythema and vesiculation in periorificial and acral areas is characteristic of necrolytic migratory erythema most often associated with pancreati c cancer. Concentric erythematous rings with trailing scale is a feature of erythema gyratum repens most commonly seen in lung cancer. Tripe palms, with or without acanthosis nigricans, are associa ted with gastric and lung cancer respectively. Q/Q(M)-482196 Report a Problem Which of the following statements about necrolytic migratory erythema is TRUE? 1

There is a low incidence of metastasis of the offending tumor at the time of diagnosis

2

Acanthosis and parakeratosis are found on routine histology

3

Serum glucagon levels are usually normal

4

Vacuolar changes are normally found on routine histology

5

The offending tumor originates from endocrine argentaffin cells Q/Q(M)-477551 Report a Problem

Which of the following statements about necrolytic migratory erythema is TRUE? 2

Acanthosis and parakeratosis are found on routine histology

 Necrolytic migratory erythema or glucagonoma syndrome syndrome is clinically characterized by periorificial and acral erythema, vesicles, pustules and erosions. A circinate pattern is often seen. Glossitis and cheilitis are features as well. The underlying tumor is an alpha-2 glucagon producing islet cell  pancreatic carcinoma, which is metastatic at the time of diagnosis in the majority of cases. Histopathological findings include dyskeratotic keratinocytes in the stratum granulosum, acanthosis and parakeratosis. Serum glucagon levels are elevated. The tumors must be resected if feasible, and intravenous somatostatin and amino acids have been used for treatment. Q/Q(M)-477551 Report a Problem

A 56-year-old female presents with longitudinal grooving of the nail plate of her index finger. Examination reveals a 4mm spongy subcutaneous nodule just distal to the DIP joint. Biopsy reveals a cyst with no true lining. What is the most common associated systemic disease: 1

Hypothyroidism

2

Bronchiolitis Obliterans Pneumonia (BOOP)

3

Hypertrophic Cardiomyopathy

4

Sporotrichosis

5

Osteoarthritis Q/Q(M)-482923 Report a Problem

A 56-year-old female presents with longitudinal grooving of the nail plate of her index finger. Examination reveals a 4mm spongy subcutaneous nodule just distal to the DIP joint. Biopsy reveals a cyst with no true lining. What is the most common associated systemic disease: 51

5

Osteoarthritis

Digital myxoid cysts are composed of mucin contained in a cystic structure without a true lining. They may have connections to the joint space, especially in patient �s with osteoarthritis, who are  predisposed to developing myxoid myxoid cysts. Historically, tuberculosis infection has also been associated with myxoid cysts but there is little current support for this relationship. Q/Q(M)-482923 Report a Problem

Patients with plexiform neuroma and NF I who also have JXG are at increased risk for developing: 1

Juvenile chronic myelogenous leukemia

2

Non-Hodgkin's Lymphoma

3

Esophageal cancer

4

Breast cancer

5

Medullary carcinoma of the thyroid gland Q/Q(M)-474942 Report a Problem

Patients with plexiform neuroma and NF I who also have JXG are at increased risk for developing: 1

Juvenile chronic myelogenous leukemia

Patient with NF 1 who have multiple juvenile xanthogranulomas are at 20 times greater risk for developing juvenile chronic myelogenous leukemia. Q/Q(M)-474942 Report a Problem

A patient comes to your office with extensive alopecia and melanocytic macules on the fingers. In the review of systems he states he has diarrhea and was told by his primary doctor that he has malabsorption symptoms. A previous colonoscopy revealed a few polyps which were removed. What kind of mutation does this patient have? 1

Autosomal recessive

2

Autosomal dominant

3

X-linked recessive

4

Sporadic

5

Mosaicism Q/Q(M)-482869 Report a Problem

A patient comes to your office with extensive alopecia and melanocytic macules on the fingers. In the review of systems he states he has diarrhea and was told by his primary doctor that he has malabsorption symptoms. A previous colonoscopy revealed a few polyps which were removed. What kind of mutation does this patient have? 4

Sporadic

Patient has Cronkhite-Canada syndrome. This is a syndrome with a sporadic PTEN mutation. Patients 52

develop GI symptoms(diarrhea, malabsorption, polyposis), weight loss and weakness. Later on they develop cutaneous changes such as melanocytic macules on the fingers, generalized hyperpigmentation and alopecia Q/Q(M)-482869 Report a Problem

Porphyria cutanea tarda may be associated with a ll of the following except: 1

Hepatitis C virus infection

2

Alcohol

3

Estrogens

4

Polyhalogenated hydrocarbons

5

Inherited deficiency of uroporphyrinogen III synthase Q/Q(M)-473929 Report a Problem

Porphyria cutanea tarda may be associated with a ll of the following except: 5

Inherited deficiency of uroporphyrinogen III synthase

A homozygous defect in uroporphyrinogen uroporphyrinogen III synthase in a cause of congenital erythropoietic  porphyria, not porphyria porphyria cutanea tarda (PCT). PCT may be caused by sporadic or familial deficiency in uroporphyrinogen decarboxylase, decarboxylase, or by any of the other causes listed above. Q/Q(M)-473929 Report a Problem

The expected histology of a biopsy take from the lesion shown in the image would 1

Increased mucin

2

Schumann bodies

3

Granulomatous infiltrate localized to the papillary dermis

4

Elastic fiber degeneration

5

Collagen degeneration Q/Q(M)-474948 Report a Problem

53

The expected histology of a biopsy take from the lesion shown in the image would 5

Collagen degeneration

The histologic findings of necrobiosis lipodica diabeticorum are: dermal granulomatous inflammation in a horizontal pattern, collagen degeneration, and normal or atrophic epidermis. Q/Q(M)-474948 Report a Problem

Paraneoplastic pemphigus is associated with all of the following underlying malignancies EXCEPT: 1

Non-Hodgkins lymphoma

2

Lung carcinoma

3

Chronic lymphocytic leukemia

4

Thymoma

5

Castleman's tumor Q/Q(M)-473958 Report a Problem

Paraneoplastic pemphigus is associated with all of the following underlying malignancies EXCEPT: 2

Lung carcinoma

Paraneoplastic pemphigus has been associated asso ciated with Non-Hodgkins lymphoma, chronic lymphocytic leukemia, thymoma, Castleman's tumor, and sarcoma. It has not been associ ated with lung carcinoma. Treatment includes management of the underlying malignancy, as well as prednisone or other immunosuppressive agents. Q/Q(M)-473958 Report a Problem Mutations in the STK11 gene encoding a serine threonine kinase are seen i n: 1

Muir-Torre Syndrome 54

2

Bannayan-Riley-Ruvalcaba Syndrome

3

Birt-Hogg-Dubbe Syndrome

4

Cronkhite-Canada Syndrome

5

Peutz-Jeghers Syndrome Q/Q(M)-473962 Report a Problem

Mutations in the STK11 gene encoding a serine threonine kinase are seen i n: 5

Peutz-Jeghers Syndrome

Peutz-Jeghers Syndrome is an autosomal dominant syndrome. 50% of patients have mutations in the STK11 gene which encodes a serine threonine kinase. It presents with peri orificial and mucosal lentigines beginning in infancy, as well as hamartomatous intestinal polyps with intussusception and  bleeding; gallbladder, pancreatic, breast, ovarian, and testicular cancer. Q/Q(M)-473962 Report a Problem

All of the following are true regarding calciphylaxis except: 1

Seen in end-stage renal disease

2

Vascular mural calcification occurs late in the process

3

High morality rate is due to sepsis

4

Association with hyperparathyroidism

5

Association with an elevated calcium: phosphate product Q/Q(M)-477374 Report a Problem

All of the following are true regarding calciphylaxis except: 2

Vascular mural calcification occurs late in the process

Calciphylaxis is a rare, life threatening disorder that is associated with end stage renal disease. Clinically, patients develop a livedoid purpura and enlarging, tender, indurated subcutaneous plaque typically on the legs or abdomen. These plaques are subject to ischemia, skin necrosis, and secondary infection, sepsis, and death. Histologic examination of calciphylaxis is characterized by the triad of small vessel mural calcivication, extravascular calcification, and vascular thrombosis. It appears that vascular mural calcification is an early and essential process in the development of calciphylaxis. Q/Q(M)-477374 Report a Problem

Which of the following is more commonly associated with ulcerative colit is as compared to Crohn �s disease? 1

Pyoderma gangrenosum

2

Oral lesions

3

Polyarteritis nodosa

4

Fistulae 55

5

Metastatic lesions Q/Q(M)-477557 Report a Problem

Which of the following is more commonly associated with ulcerative colit is as compared to Crohn �s disease? 1

Pyoderma gangrenosum

Crohn�s disease (CD) and ulcerative colitis (UC) may both have gastrointestinal tract (G IT) and cutaneous manifestations. CD lesions can involve any area of the GIT (f rom the lips to the anus), with  patchy disease normally present, whereas UC is limited to the colon and rectum and tends to be be continuous. Oral disease, fissures and fistulae, metastatic disease to the skin, and polyarteritis nodosa are more commonly associated with CD. Conversely, erythema nodosum, pyoderma gangrenosum and  pyoderma vegetans tend to be more commonly commonly associated with UC. Q/Q(M)-477557 Report a Problem

Bilateral diagonal earlobe creases may be associated with which of the following systemic diseases? 1

Gout

2

Chronic renal insufficiency

3

Interstitial pulmonary fibrosis

4

Hepatobiliary cirrhosis

5

Atherosclerotic coronary artery disease Q/Q(M)-482842 Report a Problem

Bilateral diagonal earlobe creases may be associated with which of the following systemic diseases? 5

Atherosclerotic coronary artery disease

Several studies have indicated that bilateral diagonal earlobe creases (known as �Frank �s sign�) may serve as a clinical marker for underlying coronary artery disease independent of age, particularly in persons younger than 40 years old. Q/Q(M)-482842 Report a Problem

Tripe palms are a cutaneous manifestation associated with which of the following malignancies? 1

Renal carcinoma

2

Lung carcinoma

3

Prostate carcinoma

4

Colon carcinoma

5

Pancreatic carcinoma Q/Q(M)-477338 Report a Problem

Tripe palms are a cutaneous manifestation associated with which of the following malignancies? 56

2

Lung carcinoma

Tripe palms are rugose thickening of the palms which are nearl y always associated with internal malignancy. When tripe palms are present without other cutaneous findings, the most likely internal malignancy is lung cancer. Tripe palms in association with acanthosis nigricans is most likely to be associated with gastric cancer. Q/Q(M)-477338 Report a Problem Carotenemia can be a manifestation of: 1

Porphyria

2

Hypothyroidism

3

Pretibial myxedema

4

Graves disease

5

Amyloidosis Q/Q(M)-479612 Report a Problem

Carotenemia can be a manifestation of: 2

Hypothyroidism

Reduced matabolism of beta-carotene in the diet, can result in yellowing of the skin in hypothyroidism. Treatment of porphyria with beta-carotene can res ult in carotenemia. Pretibial myxedema, Graves disease and amyloidosis do not result in carotenemia. Q/Q(M)-479612 Report a Problem A patient with end stage renal disease complains of some itchy spots on his back. Examination reveals flesh-colored papules with a central keratotic core. Acquired perforating dermatosis is more commonly seen in end stage renal disease due to what underlying disorder? 1

Diabetic nephropathy

2

Lupus nephritis

3

Mesangial glomerulonephropathy

4

IgA nephropathy

5

Medication-related renal failure Q/Q(M)-482736 Report a Problem

A patient with end stage renal disease complains of some itchy spots on his back. Examination reveals flesh-colored papules with a central keratotic core. Acquired perforating dermatosis is more commonly seen in end stage renal disease due to what underlying disorder? 1

Diabetic nephropathy

Acquired perforating dermatosis is the transepidermal elimination of altered dermal substances. The etiology is unknown, but may be related to micro-deposits of calcium with subsequent evacuation through the epidermis. It is more commonly seen in darker skin t ypes and in patients with ESRD secondary to diabetic nephropathy. 57

Q/Q(M)-482736 Report a Problem

58

View more...

Comments

Copyright ©2017 KUPDF Inc.
SUPPORT KUPDF